MKSAP13 Rheumatology

Download as doc, pdf, or txt
Download as doc, pdf, or txt
You are on page 1of 67

Rheumatology:Question 1

A 45-year-old woman with systemic lupus erythematosus who has been receiving
long-term corticosteroid therapy is evaluated in a routine office follow-up visit. She
smokes a pack of cigarettes daily.
Her blood pressure is 140/90 mm Hg. There are no abnormalities on cardiac or
pulmonary auscultation, no carotid bruits or friction rubs, and no chest wall
tenderness. Previous laboratory tests have not shown antiphospholipid antibodies.
Her total serum cholesterol concentration is 250 mg/dL.
Which of the following is the best next step in the management of this patient?
A. Measurement of serum thyroid-stimulating hormone
B. Measurement of serum creatinine
C. Measurement of fasting plasma glucose and serum lipids
D. Esophageal manometry
E. High-resolution CT scan of the chest

Rheumatology:Question 2
A 46-year-old woman with rheumatoid arthritis is evaluated because the
subcutaneous nodules that were present on her elbows have increased in size, and
she has developed some nodules on the Achilles tendons bilaterally. She has been
treated for 8 months with methotrexate in an ascending dose schedule. She is
currently taking 17.5 mg/week, along with enteric-coated aspirin, 3 g/d. This
regimen has alleviated her joint pain; her morning stiffness is limited to 35 minutes,
as opposed to the previous duration of 2 hours. On examination the nodules are not
warm, and her joints appear much improved. No other physical abnormalities are
found.
Which of the following is the most appropriate next step in this patient’s
management?
A. Biopsy a newly developed nodule.
B. Reassure the patient that these nodules occur occasionally with methotrexate
therapy.
C. Initiate antituberculosis therapy.
D. Measure fasting serum lipids, including triglycerides.
E. Measure serum uric acid.

Rheumatology:Question 3
A 27-year-old man of Arab ancestry is evaluated because of a 1-week history of
fever, rash, and muscle pain. The patient recalls that the episode began with
headache and fatigue, followed within hours by chills and fever (temperature to 38.9
°C [102 °F]). The fever has responded partially to acetaminophen, but has otherwise
persisted over the last week.
With the onset of fever, the patient developed a large, red, painful area on the right
medial thigh that has now migrated to the right calf. The patient had a similar
episode about 6 months ago that lasted 2 weeks. Within the past 3 years he has also
been treated once for unexplained fever and pleurisy, which responded to
prednisone, 20 mg/d. He was hospitalized once for a 3-week episode of unexplained
fever and abdominal pain.
As a child he had intermittent knee swelling and was diagnosed with juvenile
rheumatoid arthritis. No one in his family has had a similar illness, but the patient
states that his mother had problems with abdominal pain when she was younger,
and now has problems with intermittent redness of the eyes.
The patients temperature is 38.6 °C (101 .5 °F). He has a 5-cm erythematous
macular rash on the right calf that is exquisitely tender to palpation. Examination of
the joints shows no abnormalities.
Laboratory studies:
Leukocyte count 15,500/μL. (80% polymorphonuclear cells)
Erythrocyte sedimentation rate 120 mm/h
Serum creatine kinase 58 U/L
C-reactive protein 6.2 mg/dL (normal <0.4 mg/dL)
Rheumatoid factor 25 U/mL
Which of the following is most likely to establish the diagnosis in this patient?
A. A therapeutic trial of colchicine for 2 months
B. MRI of the right calf
C. Sequencing of exon 10 of MEFV for mutations in the pyrin protein
D. Measurement of serum ferritin
E. Sequencing of TNFRSF1A for mutations in the tumor necrosis factor receptor

Rheumatology:Question 4
A previously healthy 30-year-old man is evaluated because of a 1-year history of
right-sided lower back pain, which began after he fell off a ladder. The pain is
present when he awakens, and worsens throughout the day. It radiates down his
right posterolateral thigh to the midcalf and is somewhat worse with cough,
sneezing, and heavy lifting. The pain has been progressive and poorly responsive to
high doses of acetaminophen or ibuprofen.
The patient has a 7-month history of a persistent erythematous, scaly rash on his
elbows and knees and has noticed that his fingernails “do not look the same as they
used to.” He has no conjunctivitis, urethritis, oral or genital lesions, or joint pain.
Physical examination reveals normal, painless flexion of the lumbosacral spine;
flexion, abduction, and external rotation of the right hip do not provoke presacral
pain. The straight-leg-raising sign is negative. Neurologic examination reveals no
abnormalities.
A papulosquamous, erythematous rash is evident on the extensor surfaces of the
elbows and the knees. A number of the fingernails and toenails are pitted.
Laboratory studies, including complete blood count, erythrocyte sedimentation rate,
and C-reactive protein levels, are normal.
What is the most likely finding on CT scan of this patient’s lumbosacral spine or
sacroiliac joints?
A. A small, right-sided herniated disk at L4-L5 with minimal nerve root compression
B. Loss of disk space at multiple levels with posterior osteophyte formation and a
“vacuum disk
phenomenon” at L4-L5
C. Sclerosis and erosion with “pseudowidening” of the right sacroiliac joint
D. Anterior wedging of the L4 vertebral body

Rheumatology:Question 5
A 32-year-old Korean woman is evaluated because over the course of several
months she has lost 13.6 kg (30 Ib) and has had low-grade fevers and arthralgias.
Despite her symptoms, she has continued to work full-time in a food store. She has
recently noted back pain in the area between her shoulder blades, which she
attributed to lifting boxes at work. She also complains of achiness that develops in
her arms when she works for more than a minute stocking shelves.
On a recent evaluation in the emergency department because of back pain, she was
discovered to have a platelet count in excess of 800,000/μL and a normochromic,
normocytic anemia (hematocrit 29%, mean corpuscular volume 82 fL), but her
leukocyte count was normal. Her erythrocyte sedimentation rate was >130 mm/h.
In the office visit, she appears tired. Her pulse rate is 100/min, and her blood
pressure is 60/40 mm Hg in both arms.
What is the most appropriate next step in the management of this patient?
A. Hospitalize her for further evaluation.
B. Order blood cultures; measure antinuclear antibodies and rheumatoid factor.
C. Perform a careful physical examination; listen particularly for subclavian bruits.
D. Administer intravenous normal saline.
E. Advise the patient to take iron tablets three times a day.

Rheumatology:Question 6
A 60-year-old woman is evaluated because of a 1-year history of bilateral knee pain
and low back pain. She has some stiffness for approximately 15 minutes when she
awakens in the morning, and during the afternoon her pain is worse. As she
describes her pain, she slides her hand down the anterior thigh to her knee on the
right to show where the pain is most severe.
On physical examination, she has slight swelling and tenderness to pressure of the
distal interphalangeal joints 2-5 on both hands. All the joints are brought through full
range of motion without pain. There is slight crepitus with motion of the right knee.
Which of the following is the most likely diagnosis?
A. Rheumatoid arthritis
B. Psoriatic arthritis
C. Osteoarthritis
D. Ankylosing spondylitis
E. Osteonecrosis of the femoral condyle

Rheumatology:Question 7
A 78-year-old man is evaluated because during the past 6 hours he has developed
an acutely painful and swollen left first metatarsophalangeal joint. He has had three
such attacks during the past year. Two days ago, he had an uncomplicated
myocardial infarction and underwent percutaneous transluminal coronary
angioplasty.
On physical examination, his temperature is 38.6 °C (101 .4 °F), pulse rate is
104/min, and blood pressure is 150/95 mm Hg. He is moderately obese. He is very
uncomfortable because of a warm, dusky-red, exquisitely tender toe.
Laboratory studies:
Leukocyte count 11,000/μL
Hemoglobin 15.5 g/dL
Plasma glucose 165 mg/dL
Serum creatinine 1.7 mg/dL
Blood urea nitrogen 29 mg/dL
Serum uric acid 8.8 mg/dL
Which of the following is the best therapy for this patient at this time?
A. Oral colchicine
B. Allopurinol
C. Indomethacin
D. Intravenous colchicine
E. Prednisone

Rheumatology:Question 8
A 63-year-old woman with a long history of rheumatoid arthritis is evaluated as a
new patient. Review of old records reveals that over the past 30 years she has been
treated with salicylates, gold salts (as a result of which she developed a rash), D-
penicillamine (as a result of which she developed proteinuria), and, most recently,
methotrexate. She has required a left total knee arthroplasty, synovectomy of both
wrists, and silicone implants in eight metacarpophalangeal joints. Because of a failed
right hip arthroplasty, she has been limited to bed and chair for several years.
Despite the active disease, she has remained productive as a writer.
She has been noted to have a high-titer rheumatoid factor and the erythrocyte
sedimentation rate has never been less than 65 mm/h. Her hemoglobin has
remained stable at 8.5-9.1 g/dL.
The patient says she has increasing morning stiffness and fatigue. She has pain in
her elbows with difficulty extending them. She also has mild alopecia, shooting pains
in her forearms and hands with progressively weak hand grip, and mouth ulcers. She
states that her new orthopedic surgeon is planning a revision of her right hip
arthroplasty in hopes that she will be able to walk again. Her current medications are
methotrexate, 20 mg/week; prednisone, 7.5 mg/week; and omeprazole, 20 mg/d.
On physical examination, her vital signs are normal, and there is no evidence of
heart, lung, or abdominal abnormalities. Multiple joint deformities with arthroplasty
scars are present, along with multiple infarcts at the base of her nails and in the
finger pulp. She manifests weakness of hand grip (3/5), hyperreflexia, and
hypertonia. There are multiple subcutanous nodules on extensor surfaces.
Which of the following is the most appropriate action at this time?
A. Obtain fluorescent antinuclear antibody, anti-DNA, and anti-Sm antibody titers.
B. Test for cryoglobulins.
C. Add a tumor necrosis factor-α inhibitor to the methotrexate and decrease
prednisone.
D. Obtain images of the cervical spine.

Rheumatology:Question 9
A 46-year-old man is evaluated because of a dry, bothersome cough. He does not
smoke, has not had previous pulmonary symptoms, and has no chest pain or
dyspnea. He has no allergies. Rheumatoid arthritis was diagnosed 15 years ago. The
arthritis has been marked by swelling of the proximal interphalangeal and
metacarpophalangeal joints and some morning stiffness and virtually no pain or
limitation of motion. The patient is a farmer and rises each day at 5:00 a.m. and
works all day. His medications include enteric-coated aspirin (2.5 g/d) and a recently
prescribed angiotensin-converting enzyme inhibitor for mild hypertension. His
mother had ‘arthritis’ of some kind.
On examination, his vital signs are normal, his lungs are clear to auscultation, and
heart and abdomen are normal. He coughs frequently during the examination. He
has large hands and muscular arms. His metacarpophalangeal and proximal
interphalangeal joints bilaterally show no deformity but are swollen with boggy
synovitis and effusions. There is no ulnar deviation of the metacarpophalangeal joints
and grip strength is excellent. Subcutaneous nodules are present on both ulnae near
the olecranon.
Which of the following is most likely to be true of this patient?
A. Chest radiograph shows diffuse interstitial fibrosis.
B. A test for serum rheumatoid factor is negative.
C. Radiographs of the hand show erosions of metacarpophalangeal joints with no
periarticular
osteopenia.
D. Peripheral blood leukocyte count is less than 1500/μL, with a marked neutropenia.

E. Urinalysis shows 2 to 3+ proteinuria and no cellular elements.


Rheumatology:Question 10
A 23-year-old woman is evaluated because of recurrent episodes of fever
(temperature to 38.3 °C [101°F]) and abdominal pain. These episodes occur
approximately once every 2 months and last for 1 to 2 days. The pain, which often
begins in the periumbilical region but spreads over the entire abdomen, is usually
cramping in nature, but is sometimes a ‘dull ache,’ as she describes it.
During these episodes, the patient usually does not eat and is constipated, but at the
end of the episode she may have a loose stool. Sometimes the pain is so severe that
she stays home in bed, but at other times she is able to go to work. She went to the
emergency department once during one of these attacks, and her leukocyte count
and erythrocyte sedimentation rate were elevated. Between episodes, she feels well.
Exacerbations of the pain are sometimes associated with menses and were less
frequent 2 years ago when she was taking oral contraceptives. Last year a
colonoscopy was performed that ruled out inflammatory bowel disease. Gynecologic
examination, including ultrasound of the pelvis, was normal 3 months ago.
The patient recalls having occasional episodes of “stomach ache” as a child, and as a
10-year-old she underwent an appendectomy, but the appendix was reportedly
normal. Her mother is an Ashkenazi Jew, and her fathers family emigrated to the
United States from Sicily. A paternal uncle has had several episodes of ‘pleurisy.”
On the day of her appointment she feels well, and the physical examination is
normal. Laboratory studies are significant for a mildly elevated fibrinogen level (450
mg/dL), and a serum IgD near the upper limit of normal. A month later, a DNA test
is reported as positive for the V726A and E148Q mutations in MEFV.
Which of the following is the best treatment for this patient?
A. Prednisone, 20 mg/d for 5 days at the onset of fever or abdominal pain
B. Etanercept, 25 mg subcutaneously twice weekly
C. Colchicine, 0.6 mg twice daily
D. Interferon alfa, 3 million units subcutaneously at the beginning of each attack
E. Resumption of a low-dose-estrogen oral contraceptive

Rheumatology:Question 11
A 32-year-old woman is evaluated because of a 3-month history of joint pain
unresponsive to high doses of ibuprofen. Involved joints include the neck, left
shoulder, left knee, right ankle, and the second left toe, which are stiff or swollen in
the morning for several hours.
On physical examination, abduction of the left shoulder is restricted to 70 degrees.
The patient has full range of motion, without abnormality of the elbows, wrists, and
small joints of the hands and hips. A 10-degree flexion contracture of the left knee is
present with a moderate effusion, pain on motion, and overlying warmth. The left
ankle is diffusely swollen and warm, with pain on dorsiflexion. The second toe on the
left foot is tender and diffusely swollen in a sausage’ fashion. The patient has no
conjunctival injection, skin rash, or nail changes.
Which one of the following statements is true?
A. This patient will likely develop bilateral inflammation of the proximal
interphalangeal and
metacarpophalangeal joints and the wrists.
B. This patient is positive for rheumatoid factor.
C. This patient is positive for antinuclear antibody.
D. This patient has a history of antecedent bloody diarrhea.
E. Knee arthrocentesis will reveal intracellular monosodium urate crystals.
Rheumatology:Question 12
A 78-year-old woman is evaluated because she has had a headache for the past 8
days and has noted the onset of blurring and double vision. The double vision lasted
for approximately 15 minutes this morning but then resolved. She has lost 6.8 kg
(15 Ib) over the past 2 months. She had been taking lisinopril for mild hypertension
for 5 years.
Which of the following is the best next step in this patients management?
A. Refer her to an ophthalmologist or a neurologist.
B. Schedule a temporal artery biopsy as soon as possible.
C. Test her erythrocyte sedimentation rate and schedule a temporal artery biopsy if
it is >50 mm/h.
D. Administer sumatriptan.
E. Administer prednisone, 60 mg, immediately and schedule a temporal artery
biopsy.

Rheumatology:Question 13
A 50-year-old male surgeon is evaluated because of pain in the right inguinal area,
lower back, and hands. The inguinal pain worsens as he walks, and all pain increases
as the day progresses. On physical examination, internal rotation of the right hip
elicits the groin pain. The second, third, and fourth metacarpophalangeal joints are
swollen, warm, and tender to pressure. Radiographs show severe osteoarthritis in
the right hip.
Which of the following is the best test to confirm the diagnosis?
A. Rheumatoid factor
B. HLA-B27
C. Serum iron and iron-binding capacity
D. Fasting plasma glucose
E. Serum uric acid

Rheumatology:Question 14
A 27-year-old woman is evaluated because she had a spontaneous abortion 6 weeks
after becoming pregnant for the first time.
Laboratory studies reveal the presence of anticardiolipin antibodies. Levels of IgG
and IgA are normal. The IgM level is low-positive. Sensitive partial thromboplastin
time and modified Russell viper venom time are normal.
Which of the following would be appropriate management during this patient’s next
pregnancy?
A. Warfarin
B. Heparin, prophylactic dose
C. Heparin, therapeutic dose
D. Aspirin, 80 mg/d
E. No treatment

Rheumatology:Question 15
A 48-year-old man who underwent a successful cardiac transplantation 27 months
ago is evaluated because of recurrent pain and swelling in the joints. His graft
function is stable on a regimen of cyclosporine, mycophenolate mofetil, and low-dose
prednisone.
On physical examination, he has firm deposits around the toes consistent with tophi;
aspiration of one of these deposits yields negatively birefringent crystals.
Laboratory studies:
Leukocyte count 6800/μL
Hemoglobin 10.6 g/dL
Serum creatinine 1.6 mg/dL
Blood urea nitrogen 27 mg/dL
Serum uric acid 10.6 mg/dL
Colchicine therapy is begun, 0.6 mg/d, followed 1 week later by allopurinol, 100
mg/d. When he returns 4 weeks later, the patient has had no further pain in his
joints, but complains of weakness when he gets out of bed or arises from a chair.
Examination confirms definite proximal muscle weakness.
Laboratory studies:
Serum creatinine 1.8 mg/dL
Blood urea nitrogen 33 mg/dL
Serum uric acid 8.6 mg/dL
Serum creatine kinase 850 U/L
Which of the following is the best next step in this patients management?
A. Switch from cyclosporine to tacrolimus
B. Discontinue allopurinol
C. Increase prednisone to 60 mg/d in divided doses
D. Discontinue colchicine
E. Discontinue mycophenolate mofetil

Rheumatology:Question 16
A 26-year-old man has had progressive, active ankylosing spondylitis for 6 years
that has been refractory to maximal doses of nonsteroidal anti-inflammatory drugs,
sulfasalazine, and more recently, methotrexate (12.5 mg/week). He has severe
pains around the insertion points of tendon into bone around his knees, ankles, and
heels.
On physical examination, his neck is very painful, with limited rotation. His upward
gaze is limited to the horizon. On administration of the Schober test, points move
from 10 to 11 cm. Although the patient has only slight joint effusions, there is
marked tenderness at the insertions of quadriceps and lower leg extensor muscles,
and at the insertion of the Achilles tendon into the calcaneus bilaterally. He has a
mild anemia and a C-reactive protein level of 3.2 mg/dL. Evaluation by power
Doppler sonography shows that he has very active enthesitis at the knees, ankles,
and feet, especially the heels.
Which of the following is most appropriate management at this time?
A. Add cyclosporine to the methotrexate and sulfasalazine.
B. Begin forced stretching exercises for the cervical spine.
C. Add a tumor necrosis factor-α inhibitor to the methotrexate and sulfasalazine.
D. Obtain an MRI to document the presence of enthesitis.
E. Add prednisone, 5 mg/d, to the methotrexate and sulfasalazine after stopping the
nonsteroidal anti-inflammatory drugs.

Rheumatology:Question 17
A 62-year-old woman is evaluated because of fatigue and pain in her right knee that
started 2 weeks ago. She has had severe deforming arthritis for 23 years, which has
been treated with a succession of disease-modifying antirheumatic drugs (DMARDs),
including prednisone, gold salts, D-penicillamine, and methotrexate, in an effort to
slow progression of her disease. She has had arthroplasty of the
metacarpophalangeal joints, removal of metatarsal heads, and total joint
replacements of both knees. A small ulcer at the operative site under her right
forefoot has never healed.
In the past several years she has had periodic flares of synovitis in her shoulders,
ankles, and elbows (joints not surgically treated) that have been managed by
increasing her prednisone dosage to 15 mg/d for several weeks and changing her
basic treatment. Her current regimen is methotrexate, 12.5 mg each week;
infliximab, 4mg/kg infused every six weeks; and prednisone, 6 mg/d. Other
medications include oral hypoglycemic drugs for type 2 diabetes mellitus and
nortriptyline at night for symptoms of fibromyalgia.
Her elbows, shoulders, and ankles are not currently symptomatic. Her blood glucose
levels, tested at home, have been moderately elevated. She has not had a cough,
chest pain, bowel symptoms, or dysuria.
On physical examination, deformities without soft tissue swelling are evident in all
joints except the right knee, which has an effusion most prominent on the medial
side near the healed scar of the arthroplasty.
Which of the following would be the most appropriate management of this patient?
A. Measurement of plasma glucose and hemoglobin A1c
B. Complete blood count
C. Aspiration of the right knee effusion
D. Chest radiograph
E. Technetium-99 bone scan

Rheumatology:Question 18
A 35-year-old man is referred for evaluation of 4+ proteinuria and
hypercholesterolemia detected on a recent employment physical. When he was 8
years old, he had an episode of arthritis of the right knee requiring arthrocentesis.
Shortly thereafter he was found to have bilateral sensorineural hearing loss that has
been gradually progressive.
As an adolescent he began having 1- to 2-day episodes of chills, fever, headache,
malaise, urticarial skin rash, and pains in his arms and legs, sometimes precipitated
by emotional stress. Antihistamines have been ineffective in controlling the rash.
Recently he also noted decreased libido. His father, who has a long history of
episodic fevers, skin rash, and hearing loss, is now receiving hemodialysis.
Physical examination is remarkable for a blood pressure of 150/95 mm Hg. The
patient has bilateral hearing aids, clubbing of the digits, and bilateral pes cavus.
Laboratory studies:
Fasting plasma glucose 100 mg/dL
Total serum cholesterol 290 mg/dL
Serum albumin 2.5 g/dL
Blood urea nitrogen 30 mg/dL
Serum creatinine 2.0 mg/dL
Erythrocyte sedimentation rate 85 mm/h (Westergren)
Urine sediment shows granular and waxy casts but no cells, and urine protein is 3.5
g/24h.
Which of the following findings would be most likely in this patient?
A. Genetic testing positive for a cysteine mutation in TNFRSF1A
B. β-Microglobulin deposits present on synovial biopsy of the right knee
C. Elevated levels of mevalonic acid in the urine during a febrile episode
D. Sural nerve biopsy positive for light chain immunoglobulin deposits
E. Rectal biopsy positive for apple-green birefringent material when stained with
Congo red and viewed under polarized light
Rheumatology:Question 19
A 35-year-old woman who works in a nursing home develops acute abdominal pain
and bloody diarrhea. Many residents of the nursing home developed a similar
disorder. Several weeks later, she complains of diffuse musculoskeletal pain
involving the neck, left wrist, left knee, and low back. On examination she has
marked swelling, warmth, and loss of motion of the left wrist and left knee, as well
as pain and restricted motion of the neck and lower lumbar spine. A diagnosis of
reactive arthritis is strongly suspected.
Which one of the following statements is most likely to be true of this patient?
A. Her gastroenteritis was caused by Escherichia coli.
B. Her arthritis will resolve completely within 4 to 6 months, without complications.
C. Antibiotic treatment of the underlying infection would have prevented her
arthritis.
D. Shigella flexneri, more than any other organism associated with reactive arthritis,
is predictive of a recurrent course.
E. Neisseria gonorrhoeae, more than any other organism associated with reactive
arthritis, is predictive of a recurrent course.

Rheumatology:Question 20
A 35-year-old woman has had several episodes of paroxysmal vertigo over the past
4 months; each episode has confined her to bed for several days. The vertigo has
been accompanied on each occasion by intermittent hearing loss, first in her right
ear but more recently in her left. She thinks that her hearing in both ears is
definitely worse than before she had the vertigo. She has been evaluated twice in a
local emergency department, where she was given amoxicillin for a presumptive
diagnosis of otitis media on both occasions.
She has no pain with the vertigo. Her primary care physician treated her with
meclizine several weeks ago, but she stopped taking it because it did not seem to
help. She now presents because her eyes are red and very sensitive to the light.
Which of the following is the most likely diagnosis?
A. Menieres disease
B. Viral conjunctivitis
C. Acoustic neuroma
D. Cogans syndrome
E. Giant cell arteritis

Rheumatology:Question 21
A 78-year-old moderately obese woman is evaluated because of right hip pain that
worsens when she goes up and down stairs. She says she cannot sleep on her right
side because of pain over the hip. She has been active and in good health. On
physical examination, moving the hip through rotation, flexion, and extension does
not elicit pain, but abduction reproduces the pain minimally. There is tenderness to
pressure over the lateral aspect of the right hip.
Which of the following is the best next step in this patients management?
A. Order a radiograph of the right hip.
B. Prescribe a nonsteroidal anti-inflammatory drug.
C. Inject the painful spot with corticosteroid.
D. Refer the patient for physical therapy.
E. Advise the patient to use hot packs on the area for 20 minutes, three times daily.
Rheumatology:Question 22
A 24-year-old woman with a 5-year history of systemic lupus erythematosus is
evaluated because of weakness in both legs, which she has had for the past 36
hours. She awoke in the morning with back pain, which has grown steadily worse.
She has had some difficulty urinating during the past few hours. She played a
vigorous game of basketball the day before her weakness started, but she was not
injured during the game.
The patient has had moderate systemic lupus erythematosus, which has been
controlled with hydroxychloroquine, with no previous neurologic symptoms.
Occasional short courses of prednisone have been required to control symptoms
during flares.
On physical examination, she has weakness of the hip flexors and knee extensors,
but she can stand and walk. Range of motion in the spine is normal, and there is no
local tenderness. Lower-extremity sensation is intact. Rectal examination reveals
diminished sphincter tone. Deep-tendon reflexes are intact, and there are three
beats of clonus at the left ankle.
Which one of the following studies is the most appropriate at this time?
A. MRI of the spine
B. CT of the spine
C. Radiographs of the spine
D. Analysis of the cerebrospinal fluid
E. Measurement of the antiphospholipid antibody level

Rheumatology:Question 23
A 68-year-old woman has had four or five episodes of joint pain and swelling, lasting
3 to 8 days, involving the right knee and left elbow. She is asymptomatic between
attacks, and sulindac, 200 mg twice daily, has usually relieved the symptoms. Her
most recent episode was 4 months ago.
On physical examination, none of her joints is swollen or tender, but there is marked
crepitus on extension of the knee. She also has a positive bulge sign over the left
knee and pain on full extension of the left elbow.
Which one of the following tests would confirm the diagnosis?
A. Arthrocentesis of the knee and laboratory analysis of the synovial fluid
B. Measurement of serum uric acid
C. Measurement of serum rheumatoid factor
D. Radiograph of the knee
E. MRI of the knee with gadolinium contrast

Rheumatology:Question 24
A 32-year-old woman is evaluated because of a 5-year history of Raynauds
phenomenon and thickened skin over the fingers. Two years ago, she developed
small patches (1 to 2 mm in diameter) of calcinosis on the distal fingers and dorsum
of one hand, which have persisted. She reports mild dyspnea when she climbs stairs.
She takes a proton pump inhibitor for reflux esophagitis.
Physical examination reveals sclerodactyly, with one digital tip ulcer, minimal
calcinosis, and two telangiectasias on her face, without limitation of oral opening.
Lung and heart examination are normal. Chest radiograph reveals no infiltrates or
interstitial fibrosis in the lungs and a normal heart size. Results of routine laboratory
tests are normal, including renal function. Echocardiogram suggests mild right
ventricular diastolic dysfunction. The diffusing capacity for carbon monoxide (DLCO)
is 44% of normal.
Within the next 5 years, she is at high risk for developing which of the following?
A. Fine, dry crackles indicative of pulmonary fibrosis
B. Renovascular hypertension with hyperreninemia
C. Sclerodermatous changes on the forearms and face
D. Pulmonary hypertension
E. Esophageal varices

Rheumatology:Question 25
A 19-year-old female college freshman is evaluated in her student health clinic for a
3-day history of fever (temperature to 38.3 °C [101 °F]), abdominal pain, arthralgia,
and skin rash. The episode began with headache and malaise, and within hours she
developed chills and fever. At about the same time, she developed a rash covering
most of her body. She also had abdominal pain with loose stools, and “achiness” of
both knees and ankles.
The patient has had similar episodes, which usually last about 5 days, about six
times a year since she was a young child. She has a 5-year-old sister who has a
similar history; her parents and her 12-year-old brother are healthy. Her family of is
mixed Dutch and German ancestry.
Eight years ago, her pediatrician told her that she might have familial Mediterranean
fever. For the past year she has been taking colchicine, 1.2 mg/d, but it has not
helped much.
Physical examination is remarkable for a temperature of 38.1 °C (100.6 °F) (she had
just taken acetaminophen before coming to the clinic), a diffuse maculopapular skin
rash affecting most of her body, including the palms and soles, and tender cervical
lymphadenopathy. She has moderate abdominal tenderness on deep palpation,
without rebound. Joint examination is normal. Laboratory studies include a leukocyte
count of 13,800/μL, erythrocyte sedimentation rate of 62 mm/h and trace protein on
urinalysis.
Which of the following would be most appropriate for this patient?
A. Increase the dose of colchicine to 2.4 mg/d.
B. Administer colchicine, 1 mg intravenously in the clinic for 3 days.
C. Schedule a rectal biopsy to rule out amyloidosis.
D. Order genetic testing for mutations in the mevalonate kinase gene.
E. Schedule ophthalmologic examination to rule out uveitis.

Rheumatology:Question 26
A 50-year-old man is evaluated because he has increasing neck pain after a fall from
the second rung of a ladder 5 days ago. He has a long-standing history of
generalized stiffness and mild pain in the lower back and neck for which he has not
sought medical attention, but his medical history is otherwise noncontributory. He
has no weakness or numbness in any extremity. He has not had loss of bowel or
bladder function or other neurologic symptoms.
On physical examination, he appears healthy, but has moderate discomfort due to
neck pain. Gentle range-of-motion testing of the neck shows restriction of extension
greater than flexion of the cervical spine, with attempted flexion resulting in
increasing neck pain. There is marked paraspinous muscular tightness bilaterally and
dorsal kyphosis with marked loss of motion of the thoracic spine and significant
limitation of chest expansion. There is marked loss of flexion of the lumbar spine,
and a Schober test reveals only 2 cm of spinous excursion.
Neurologic examination shows intact muscle strength proximally and distally in upper
and lower extremities, slight dysesthesias to pinprick of the fingers and toes, and
generalized hyperreflexia (3+ to 4+) in the upper and lower extremities. Babinskis
sign is present on the right but absent on the left.
Which of the following is the best next step in this patients management?
A. Prescribe analgesics and muscle relaxants.
B. Recommend physical therapy for the neck and paraspinous musculature.
C. Prescribe indomethacin for treatment of pain secondary to axial skeletal disease in

spondyloarth ropathy.
D. Immobilize the neck and refer the patient to the emergency department for
cervical spine radiographs or CT.
E. Reassure the patient that within a few weeks his pain will resolve without specific
therapy.

Rheumatology:Question 27
An 18-year-old man from Lebanon who is a freshman in college is evaluated because
he has noted significant problems with memory, and his ability to write compositions
has deteriorated markedly. His academic performance generally has declined over
the course of the year. He states that he has had intermittent heartburn and lack of
energy. Several weeks ago, he had severe pain in his shoulder, which was made
worse with his arm in abduction while he was painting the walls in his apartment.
The pain improved when the patient rested his arm and applied ice packs then warm
compresses. An evaluation has included an MRI of the brain, which showed
multifocal subcortical T2 hyperintensities in both parietal regions. A lumbar puncture
revealed a cerebrospinal fluid protein level of 81 mg/dL, but no oligoclonal bands and
no cells. Complaints of mild visual blurring in one eye led to an evaluation by an
ophthalmologist, who detected evidence of posterior uveitis in both eyes.
For the past several years, the patient has had frequent oral ulcers, often several at
a time. He has also had one episode of a scrotal ulcer several months ago, at a time
when he was not sexually active.
Which of the following is the most likely diagnosis?
A. Behçet’s disease
B. Giant cell arteritis
C. Multiple sclerosis
D. Microscopic polyangiitis
E. Polyarteritis nodosa

Rheumatology:Question 28
A 72-year-old retired lawyer is evaluated because of swelling of his right knee. He
has mild osteoarthritis and stays active swimming and playing tennis and golf.
On physical examination, he has an effusion in the right knee. At a point in the mid-
thigh (measured from the superior border of the patella), the right thigh is 4 cm
smaller in diameter than the left. There is crepitus with right knee flexion. The knee
is not warm.
Which of the following is the best next step in the management of this patient?
A. Order radiographs of both knees and treat the patient with ibuprofen.
B. Order radiographs of the right hip and right knee.
C. Aspirate the joint fluid in the right knee and refer the patient for physical therapy.
D. Aspirate the joint fluid in the right knee and obtain blood cultures.
E. Treat the patient with celecoxib, and advise him to stay off his feet for 48 hours.

Rheumatology:Question 29
A 45-year-old woman fell forward, landing on the front of her right knee. A
radiograph taken at an outside facility shows a joint effusion but no fracture. She is
unable to bear weight on her right leg.
On physical examination, she has an ecchymosis and some swelling over the patella,
with point tenderness over the mid-patella. The knee lacks 10 degrees of full
extension, and flexes to 60 degrees. Aspiration of the knee produces 10 mL of
bloody synovial fluid, with a hematocrit of 6%.
Which of the following is the most likely diagnosis?
A. Anterior cruciate ligament tear
B. Fracture of the patella
C. Osteogenesis imperfecta
D. Traumatic arthrocentesis
E. Pigmented villonodular synovitis

Rheumatology:Question 30
A 74-year-old woman is evaluated because of pain in her right hip. She says that it is
worse when she lies on her right side in bed and when she crosses her right leg over
her left.
The patient has had low-grade systemic lupus erythematosus (SLE) for 20 years,
which has been treated intermittently with low-dose prednisone and
hydroxychloroquine. She has had no evidence of renal disease, and tests for
anticardiolipin antibodies have been negative. She was involved in a serious
automobile accident 14 years ago that necessitated transfusions, and 4 years ago,
cirrhosis of the liver was diagnosed. During the past year, her SLE has been stable,
with only occasional joint pains. She has not bled from varices, although she has had
ascites.
On physical examination, she is afebrile. All joints are normal except for the right
hip. She has tenderness on lateral pressure at the hip and no tenderness on groin or
buttock pressure. She has no pain on full flexion and rotation of the leg at the hip
through slow passive range of motion. She has moderate pain in the lateral hip area
on adduction, which is limited to 40 degrees, and on forced abduction.
Which of the following is the most likely diagnosis?
A. Trochanteric bursitis
B. Avascular necrosis of the femoral head
C. Osteoarthritis of the hip
D. Adductor muscle strain
E. Septic arthritis of the hip

Rheumatology:Question 31
A 47-year-old woman developed rheumatoid arthritis when she was 29 years old that
was limited at first to her hands and wrists and spread to involve her knees when
she was 35 years old. It has involved no other joints. A rheumatoid factor test was
positive in high titer when first measured when the patient was 30 years old.
On an initial regimen of intramuscular gold therapy and subsequent methotrexate
and hydroxychloroquine, her arthritis has been inactive, and her only medication now
is prednisone, 5 mg/d, and supplemental calcium and vitamin D.
A complication, however, has been progressive neutropenia. For the past 10 years
she has had an enlarged spleen, which has been documented by CT scan. She has
developed several small ulcers on her lower legs and often has apthous ulcerations.
In the past 2 years, she has had four bouts of community-acquired pneumonia,
treated aggressively and successfully with antibiotics. Because of these infections
and the presence of circulating neutrophils of less than 200/μL, a trial of granulocyte
colony-stimulating factor and later, granulocyte-macrophage colony-stimulating
factor was initiated. Both stimulating factors helped the neutropenia but caused
severe and incapacitating flares of her rheumatoid arthritis.
On this visit, she is free of active infection but has apthous ulcers and gingivitis.
There is some ulnar deviation at the metacarpophalangeal joints but no synovial
proliferation, and she complains of no arthritis symptoms.
Laboratory studies:
Hemoglobin 8.6 g/dL
Leukocyte count 1400/μL (12% polymorphonuclear cells, 60% small
lymphocytes, 4% eosinophils, 24% large granular lymphocytes)
Platelet count 43,000/μL
Erythrocyte sedimentation rate 23 mm/h
C-reactive protein 0.7 mg/dL
Serum globulin 4.2 g/dL (normal immunoelectrophoresis)
Serum albumin 3.2 g/dL
Rheumatoid factor 180 IU/L
Fluorescent antinuclear antibodies Positive 1:1280 (speckled pattern)
Anti-DNANegative
Liver and kidney function test results are normal (including urinalysis).
Which of the following is the most appropriate therapy at this time?
A. Schedule splenectomy and administer pneumococcal vaccine.
B. Administer small weekly doses of granulocyte-macrophage colony-stimulating
factor.
C. Resume methotrexate therapy.
D. Initiate therapy with etanercept, 25 mg subcutaneously twice weekly
E. Administer plasmapheresis

Rheumatology:Question 32
A 27-year-old man is evaluated in the emergency department because of hives that
break out within an hour of exposure to cold weather; he was out shoveling snow
earlier this morning, and states that he feels an attack coming on. The patient lived
in San Diego all of his life but moved to Buffalo 3 months ago. As the weather
became colder, he found that going outside induced headache, drowsiness, chills,
fever, extreme thirst, joint stiffness, and hives covering most of his body. The rash
usually begins on the face and extremities, and then generalizes to the rest of the
body over several hours.
Sometimes he experiences watering of his eyes and blurred vision, and if he looks in
the mirror at these times his eyes look ‘bloodshot. As the weather has become
colder, the length of time needed to provoke an attack has decreased. Initially there
was approximately a 2-hour delay from the beginning of cold exposure to the onset
of symptoms, but this interval has shortened. Most episodes last 10 to 12 hours.
Drinking cold beverages or picking up frozen foods in the grocery store do not induce
symptoms. The patient is of Indonesian ancestry; his father and one of his two
brothers have experienced similar, although much milder symptoms when going into
air-conditioned rooms.
On physical examination, the patients temperature is 38.1 °C (100.6 °F). He has a
hive-like rash on his face and arms and conjunctivitis. Laboratory studies include a
leukocyte count of 24,000/μL, an erythrocyte sedimentation rate of 77 mm/h, and a
C-reactive protein level of 6.0 mg/dL (normal <0.4 mg/d L).
Which of the following would be most useful in treating this patient?
A. Continuous treatment with a combination of H1- and H2-specific antihistamines
B. Measurement of serum cryoglobulin levels
C. Administration of a nonsteroidal anti-inflammatory drug to control the fever and
joint stiffness
D. Use of an epinephrine syringe at the onset of an attack
E. Administration of the ice-cube challenge test when the patient is asymptomatic

Rheumatology:Question 33
A 34-year-old man with no known major medical problems seeks a second opinion
concerning management of toe pain. One year ago, the patient developed redness,
warmth, and swelling in the second toe of his left foot. He saw an orthopedist who
took a radiograph of the area and found a large erosion adjacent to the
interphalangeal joint extending into the shaft of the bone. An MRI was read as
showing marrow edema and changes consistent with osteomyelitis.”
The patient was treated with an extended course of parenteral broad-spectrum
antibiotics with little improvement, and the toe was ultimately amputated. Cultures
at the time of the amputation did not reveal any organisms, and the changes on
pathologic examination were nonspecific. Within 6 months, he developed similar
findings in the adjacent third toe. His physician once again recommended an MRI,
which showed erosive changes involving that digit as well as several mid-tarsal
bones. Intravenous antibiotics were once again recommended, but this time he
refused. He states that he has had intermittent redness and pain in his left eye for a
long time, for which he has seen an ophthalmologist and has used topical eye drops,
but he does not know the specific diagnosis. He also has frequent dysuria, which has
been attributed to “prostatitis.” Examination of his feet shows generalized redness
and “sausage swelling” of the third left digit, which is tender to touch and painful on
motion. Which one of the following statements is true?
A. HLA-B27 testing will establish this patients diagnosis.
B. This patients original diagnosis of osteomyelitis was correct, and the disease has
spread to adjacent bones.
C. This patient is likely to complain of low back pain, worse with activity as the day
progresses.
D. This patient has a scaly rash on his palms and soles, onycholytic nail changes, and
lesions on the glans penis.
E. This patients ocular disease is due to posterior uveitis.

Rheumatology:Question 34
A 29-year-old woman is evaluated because of a 4-year history of skin ulcers on her
lower extremities. The ulcers usually begin with tender, erythematous lesions that
then ulcerate and are resistant to treatment.
Two ulcers (one on each leg, just superior to the medial malleolus) have been as
large as 5 or 6 cm in diameter, and the others, also confined to the distal legs and
dorsal surfaces of the feet, have been 1 to 2 cm in diameter. The borders of the
ulcers are scalloped. A previous punch biopsy of the skin allegedly showed
thrombotic lesions in small blood vessels of the dermis. Results of antinuclear
antibody testing have been negative on many occasions. Treatment has consisted of
careful local wound care and aspirin and dipyridamole therapy. Because of
recurrence of the ulcers despite this therapy, consideration is being given to the
institution of warfarin therapy. The physical examination is remarkable for the ulcers,
and also for a diffuse livedo reticularis pattern of skin discoloration and wasting of
the web space between the thumb and index finger on the left hand. Her ability to
grasp with that hand is minimal. Her right hand grip also appears weak, but there is
no obvious muscle wasting. She has a right foot drop.
Which of the following diagnoses is the most likely cause of this patients symptoms?
A. Lymphoma, with a paraneoplastic syndrome
B. Takayasus arteritis
C. Systemic lupus erythematosus
D. Polyarteritis nodosa
E. Kawasaki’s disease

Rheumatology:Question 35
A 31-year-old woman who recently returned from a Caribbean vacation is evaluated
because of a 2-week history of malaise, ear swelling, aching of her wrists and
metacarpophalangeal and proximal interphalangeal joints, and vertigo. She has no
history of gastrointestinal or respiratory problems.
On physical examination, she is afebrile. She has some erythema of the malar areas
and pain on passive motion of the wrists. The pinnae of her ears are swollen, red,
and warm. Otoscopic examination is normal.
Which of the following is the best test to confirm a diagnosis?
A. Antinuclear antibody (ANA) test
B. Rheumatoid factor test
C. Biopsy of ear cartilage
D. Radiograph of the wrist
E. Stool culture

Rheumatology:Question 36
A 55-year-old woman who has had rheumatoid arthritis for 10 years is evaluated
because of severe pain in the left shoulder that developed over the course of the
previous day. In recent months, her disease has been poorly controlled on a regimen
of methotrexate, hydroxychloroquine, and low-dose prednisone. She has
approximately 90 minutes of morning stiffness.
On physical examination, her temperature is 36.8 °C (98.2 °F), her pulse rate is
82/min, and her blood pressure is 110/70 mm Hg. She has moderate tenderness of
the small joints of her hands and of both wrists. Her left shoulder is warm and very
tender; she can move it only slightly before being limited by pain.
What is the best next step in this patient’s management?
A. Orthopedic consultation for possible shoulder arthroplasty
B. Aspiration of the shoulder
C. Radiography of the shoulder
D. MRI contrast arthrography of the shoulder
E. Physical therapy for the shoulder

Rheumatology:Question 37
A 64-year-old man is evaluated because of bilateral ankle pain that has progressively
worsened over the course of a month. Elevating his feet alleviates the discomfort,
and he sleeps with his lower legs propped up on four pillows. He has had no history
of arthritis. He took levothyroxine, 0.15 mgld, each day for many years until the past
2 weeks when his prescription ran out.
On physical examination, both lower legs are slightly warm, but not red, and there is
1+ pitting edema that begins 6 cm above the malleoli. The tibias are tender to
compression in the edematous regions. No ankle effusions can be palpated. Results
of routine laboratory tests are normal except for a leukocyte count of 11,300/μL and
an erythrocyte sedimentation rate of 46 mm/h. A radiograph of both lower legs and
feet shows mild soft tissue swelling in the tissues around the left ankle joint and
minimal but definite periosteal new bone formation on the left medial tibia, 2 to 5 cm
above the ankle joint.
Which of the following is the best next step in this patients management?
A. Prescribe nonsteroidal anti-inflammatory drugs.
B. Order thyroid function tests.
C. Order a chest radiograph.
D. Administer a tuberculin skin test.
E. Order a radionuclide bone scan of both lower legs.

Rheumatology:Question 38
A 60-year-old man with rheumatoid arthritis is evaluated because of the onset in the
past 2 weeks of fever, a dry cough, and increasing shortness of breath. He has been
treated for 5 months with methotrexate, 12.5 mg/week; folic acid, 1 mg/d; and
prednisone, 5 mg/d. He was active, bicycling 20 miles each day, until his symptoms
began. His wife has been recovering from pneumonia believed to be caused by a
Pseudomonas organism. He has had no recent travel history and has no pets or
known allergies.
On physical examination, he appears anxious. His temperature is 38.5 °C (101 .3 °F)
and his pulse rate is 110/min. On examination of the lungs, diffuse fine crackles are
heard. There is no pericardial friction rub or S3, and there are no murmurs.
Abdominal and neurologic examinations are normal. He has synovial swelling without
warmth in both wrists and metacarpophalangeal joints. Chest radiograph shows
diffuse interstitial infiltrates. Which of the following is the most likely diagnosis?
A. Bacterial pulmonary infection
B. Methotrexate-induced lung disease
C. Diffuse interstitial fibrosis
D. Reactivation of latent tuberculosis
E. Coronavirus infection

Rheumatology:Question 39
A 38-year-old man is evaluated because of a long history of leg ulcerations and
arthritis. The patient first presented with arthritis at the age of 4 years, when he
developed a destructive sterile abscess of the left hip joint that required incision and
drainage and eventually bone grafting. As a teenager he developed a painful, swollen
right elbow that eventually required drainage and healed with a significant loss in
mobility. At the age of 28 years, following mild trauma to his right knee, he
developed a large effusion; cultures at the time were negative. The patient was
treated with prednisone, 60 mg/d for several weeks, which induced marked
improvement without significant sequelae.
As a teenager, he had severe cystic acne of the face and back. As an adult he began
to have pustules on his leg that would break down spontaneously and rapidly enlarge
to form leg ulcers. Cultures were repeatedly negative, and treatment with antibiotics
was ineffective. The leg ulcers have become more chronic, and the current ulcer on
his right leg has been present for over a year. The patient also developed a sterile
abscess after a tetanus immunization and once developed a pustule at the site where
his blood had been drawn. He has no history of oral or genital ulcers, conjunctivitis,
eye pain, photophobia, thrombophlebitis, or blood in his stool. The patient is of Irish
and Italian ancestry and reports that both his mother and maternal grandfather had
similar symptoms when they were living. The physical examination is remarkable for
a 5-cm ulceration on the right lower leg, with blood and pus covering most of the
open area. Extensive scarring is evident on his face and back, and movement is
limited in his right elbow and left hip. Laboratory studies show a leukocyte count of
16,200/μL, hematocrit of 34%, erythrocyte sedimentation rate of 68 mm/h, and C-
reactive protein level of 10.2 mg/dL (normal <0.4 mg/dL). Radiographs of both the
left hip and right elbow show severe joint destruction.
Which of the following studies is most likely to yield the diagnosis in this patient?
A. DNA sequencing for mutations in the NOD2 gene
B. Screening for the HLA-B51 antigen
C. Screening for HLA-DR1 and DR4 antigens
D. DNA sequencing for mutations in the PSTPIP1 gene
E. Bone marrow aspiration and biopsy

Rheumatology:Question 40
A 50-year-old woman is evaluated because of a 5-year history of diffuse skin lesions,
joint pain, and back pain. Her rash was initially diagnosed as psoriasis. She has tried
only topical therapies, which were modestly helpful. One year ago, she developed
redness, pain, and swelling in several fingers, her right knee, and left ankle. The
symptoms, which have continued, are worse in the morning and have been
progressive, leading to functional difficulty. She has also recently had severe right-
sided lower back pain, especially on arising, which improves as the day goes on.
Physical examination reveals a typical papulosquamous eruption involving the
extensor surfaces of the elbows and knees and more than 50% of the back and
chest. Most of her nails are pitted. Joint examination shows erythema and swelling of
the right first and third distal interphalangeal (DIP) joints, the right second proximal
interphalangeal (PIP) joint, and the left second DIP and fourth PIP joints. There is a
large, warm right knee effusion with a 20-degree flexion deformity. Warmth,
swelling, and pain on motion are present in the left medial malleolus.
What is the most appropriate treatment for this patient?
A. A nonsteroidal anti-inflammatory drug
B. Intensive topical corticosteroids for the psoriatic skin lesions
C. A nonsteroidal anti-inflammatory drug followed by hydroxychloroquine
D. A nonsteroidal anti-inflammatory drug followed by escalating doses of
methotrexate
E. A nonsteroidal anti-inflammatory drug followed by etanercept

Rheumatology:Question 41
A 28-year-old man who works as a software designer is evaluated because he
developed painful fingers in the cold weather several weeks ago and has continued
to have pain in several fingertips of both hands intermittently. The arch of his right
foot also began to ache when he walked and eventually he noted similar symptoms
in his left foot. Over the past week his left second toe has turned black, and the third
and fourth toes are purple. The patient has no risk factors for endocarditis (no
history of rheumatic heart disease, no recent dental work, no intravenous drug use).
He smokes two packs of cigarettes a day.
Findings on physical examination include splinter hemorrhages under several
fingernails; pulses are absent in both feet. Numerous blood cultures are negative. An
angiogram shows a normal proximal aorta but corkscrew collateral vessels and
abrupt vascular cutoffs at the level of the wrists and ankles.
What is the most appropriate treatment for this patient?
A. Smoking cessation
B. Pulse methylprednisolone, followed by prednisone, 1 mg/kg daily
C. Daily cyclophosphamide and prednisone
D. Hospitalization for administration of heparin, followed by warfarin
E. Consultation with a hematologist for possible use of a thrombolytic agent

Rheumatology:Question 42
A 77-year-old man is evaluated because he has been told by another physician that
he needs knee joint replacement surgery. He has been healthy and active all his life,
having been very athletic in high school and college. He has given up jogging. Two
years ago, he cut down on playing tennis from five to three days a week because of
pain and swelling in his right knee; radiographs at the time showed moderate medial
compartment narrowing, absence of the patellofemoral space, and small
osteophytes.
He now plays tennis once a week and golf three times a week. He is able to go up
and down the stairs in his house and put on his slacks and shoes and socks with
minimal discomfort. He is not awakened at night by pain. Radiographs of the knee
now show further medial compartment narrowing and more osteophyte formation.
MRI shows partial tear of the posterior horn of the medial meniscus. He asks whether
he should have his right knee replaced.
Which of the following is the best advice for this patient?
A. Have the joint replacement surgery.
B. Have arthroscopic surgery to repair the meniscal tear.
C. See a physical therapist for exercises to strengthen all lower extremity muscles.
D. Have arthroscopy to shave fibrillated cartilage and lavage the joint.
E. Reinstitute a mild jogging program.

Rheumatology:Question 43
A 28-year-old pregnant woman, who previously had a single early first-trimester
pregnancy loss, is evaluated in a routine office visit. An anticardiolipin antibody assay
done after she suffered her first pregnancy loss was moderately positive, and there
was no lupus anticoagulant. Her obstetrician suspects a hypercoagulable state and
has started her on subcutaneous unfractionated heparin (twice daily) during her
current pregnancy. An anticardiolipin assay and lupus anticoagulant testing are
ordered and yield the following results.
Laboratory studies:
Anticardiolipin IgG Low-positive range
Anticardiolipin lgM Normal
Anticardiolipin IgA Normal
Sensitive partial thromboplastin time 45 sec (slightly prolonged)
Modified Russell viper venom time36 sec (normal <37 sec)
Which one of the following is the correct interpretation of these laboratory tests?
A. Because only one of the two clotting times is abnormal, there is no lupus
anticoagulant.
B. The patient has neither a lupus anticoagulant nor antiphospholipid antibody
syndrome.
C. The patient has both a lupus anticoagulant and antiphospholipid antibody
syndrome.
D. The lupus anticoagulant tests cannot be interpreted.
E. The partial thromboplastin time proves the presence of a lupus anticoagulant.

Rheumatology:Question 44
A 27-year-old man is evaluated because of a 1-week history of pain in the left
lumbar region, radiating into the left buttock and down the back of the leg. He has
difficulty bending over or bearing weight on the leg. He is an injection drug user and
is positive for HIV.
On physical examination, his temperature is 36.1 °C (97.0 °F), pulse rate is 76/min,
and blood pressure is 135/85 mm Hg. There is focal tenderness in the left buttock.
The straight-leg-raising test is positive. There is pain with pelvic compression.
Which of the following is the best next step in this patients management?
A. Bone scan
B. MRI of the lumbar spine
C. MRI of the pelvis
D. Radiograph of the pelvis
E. Aspiration of the sacroiliac joint

Rheumatology:Question 45
A 68-year-old man has severe osteoarthritis of the hips and knees. Predisposing
factors include a strong family history of osteoarthritis of the hip, obesity, and coax
vara. He has taken nonsteroidal anti-inflammatory drugs and
glucosamine/chondroitin sulfate. He has had a course in each knee of injections of
high-molecular-weight hyaluronic acid. His religious denomination opposes blood
transfusions, and he has avoided all discussions of surgery for his arthritis.
Over the past year he has had progressive night pain in both hips and knees that
interferes with sleep. He estimates that at most he gets 3 to 4 hours sleep each
night and is exhausted when he arises each morning. He cannot tolerate
conventional sleeping medications because of intolerable hangovers. Combinations of
hydrocodone/acetaminophen produce constipation. He is becoming increasingly
irritable and knows that sleep deprivation is compromising his intellectual function.
He has no allergies and takes glyburide for type 2 diabetes mellitus and an
angiotensin-converting enzyme inhibitor in addition to his various medications for his
osteoarthritis. He has had two episodes of gastrointestinal bleeding, presumably
from aspirin. His liver function is normal, but his serum creatinine level has been
rising for 10 years to its current value of 1.9 mg/dL.
On physical examination, he is afebrile. His pulse rate is 94/min and regular, and his
blood pressure is 154/91 mm Hg. He has bony proliferation at both knees, 30 flexion
contractures of the knees, hip contractures, and intense pain on the limited motion
that he has in both hips and knees. Neurologic examination is normal.
A diagnosis of severe osteoarthritis is made, with a secondary pain syndrome that
has produced severe sleep deprivation.
Which of the following is the best next step in this patients management?
A. Refer him to a sleep clinic.
B. Prescribe a single daily dose of low-dose long-acting morphine.
C. Prescribe full doses of glucosamine supplement, 2000 mg/d.
D. Prescribe rofecoxib.
E. Prescribe acetaminophen in maximal doses.

Rheumatology:Question 46
A 46-year-old man is evaluated because for the past 2 weeks he has felt ill, with
upper respiratory tract stuffiness, ear fullness, fatigue, and myalgias; this morning
he awoke to find red spots over his buttocks and legs. He has been evaluated by his
primary care physician, who gave him a prescription for amoxicillin, which he took
for 10 days and then stopped because he was having diffuse stomach pain.
The patient states that his urine has appeared abnormally dark, but he has no
dysuria. He is taking no medications, has had no serious illnesses (no cardiac or
respiratory disease), and has no allergies. On physical examination, the only
abnormalities noted are palpable, nontender, purpuric papules that range in diameter
from 1 to 5 mm.
Which of the following diseases is most likely to be the cause of this patient’s
presentation?
A. Microscopic polyangiitis
B. Acute lymphocytic leukemia
C. Temporal arteritis
D. Churg-Strauss vasculitis
E. Hypersensitivity vasculitis (cutaneous leukocytoclastic angiitis)

Rheumatology:Question 47
A 66-year-old woman is evaluated because of right knee pain of 4 weeks duration.
Although the knee is stiff for 20 to 30 minutes in the morning, she does not have
much pain at work. Walking up the stairs in her house, however, causes a good deal
of pain, which is not relieved by ibuprofen (600 mg three times daily) or by
acetaminophen (1000 mg three times daily). Knee radiographs done 6 weeks ago
show mild medial compartment osteoarthritis bilaterally.
On physical examination, she is somewhat overweight, there is coarse crepitus with
flexion/extension of the right knee. Both knees are in slight varus angulation (bow-
legged). On palpation there is tenderness along the joint margins of both knees and
exquisite tenderness to digital pressure at the medial upper tibia on the right. In
addition, with the patient’s right knee semiflexed, palpation along the medial
semimembranous tendinous (hamstring) edge of the thigh elicits pain when the
examining fingers meet the tibia.
Which of the following would be the best management of this patient?
A. Increase the ibuprofen dose to 800 mg, four times daily.
B. Initiate prednisone, 15 mg/d.
C. Inject the tender point with a depot corticosteroid.
D. Obtain an MRI of the knees.
E. Refer the patient for physical therapy exercises and a knee brace.

Rheumatology:Question 48
A 39-year-old woman with systemic lupus erythematosus who has been receiving
long-term corticosteroid therapy is evaluated in the emergency department because
of substernal chest pain of several hours duration. The pain is not positional or
pleuritic.
On physical examination, there is no chest wall tenderness, and lung fields are clear
to auscultation. On cardiac examination, there is tachycardia. Previous laboratory
tests have not shown the presence of antiphospholipid antibodies. Electrocardiogram
shows anterolateral T-wave changes. Arterial blood gases are normal.
What is the best initial diagnostic test?
A. Measurement of serum troponin or serum creatine kinase MB
B. Ventilation-perfusion scan
C. Echocardiography
D. Pulmonary function tests
E. High-resolution chest CT

Rheumatology:Question 49
A 46-year-old man is evaluated because of pain and swelling in his right knee. He is
no longer able to play soccer because his knee gives way and “locks” on him.
He has hypertension, for which he is being treated with lisinopril, 20 mg/d, and
hydrochlorothiazide, 50 mg/d. He smokes 1 pack of cigarettes daily, and consumes 6
to 12 cans of beer on the weekends.
On physical examination, he weighs 99.8 kg (220 Ib). His temperature is 37.2 °C
(98.9 °F), pulse rate is 84/min, and blood pressure is 180/90 mm Hg. There is some
patellofemoral crepitus, but no effusion of the knee. There is tenderness along the
joint line medially. Small nodules are present in both olecranon bursae.
Laboratory studies:
Hemoglobin 15.6 g/dL
Blood urea nitrogen 18 mg/dL
Serum creatinine 1.1 mg/dL
Serum uric acid 10.5 mg/dL
Which of the following is the best next step in this patients management?
A. Discontinue hydrochiorothiazide.
B. Aspirate the knee and inject 40 mg of triamcinolone hexacetonide.
C. Initiate therapy with allopurinol.
D. Refer him for arthroscopy.
E. Initiate celecoxib therapy.

Rheumatology:Question 50
A 38-year-old black man is brought to the emergency department after being found
unconscious. After being admitted to the hospital and resuscitated with fluids he
awakes complaining of severe joint pain, which is relieved in most joints by
narcotics. The right hip remains painful, however, and on examination has a very
limited range of motion. Appropriate tests reveal that the patient is homozygous for
sickle cell anemia. A radiograph of the hip is likely to reveal which of the following?
A. Avascular necrosis within and periosteal elevation of the femoral head
B. Cartilage and bone erosions similar to those seen in severe rheumatoid arthritis
C. Chondrocalcinosis with osteoarthritis of the hip
D. Subperiosteal bone resorption of the femoral neck
E. Periosteal new bone formation around the greater trochanter

Rheumatology:Question 51
A 42-year-old woman has had arthralgias and occasional purple spots on her legs for
several years. She now presents with florid small, nonblanchable, palpable lesions on
her legs and obvious synovitis of multiple small joints. Some of the skin lesions have
tiny centers of necrosis. Radiographs of the hands and wrists show no erosions.
In the past, she has been diagnosed at different times with rheumatoid arthritis and
with systemic lupus erythematosus, because results of testing for rheumatoid factor
are markedly positive and she has antinuclear antibodies. In addition, on several
occasions, her serum C4 level has been found to be as low as 4 mgldL (normal: 11
to 26 mg/dL) at times when her C3 has been normal. Twenty years ago, she had a
complicated but safe delivery of her only pregnancy, a breech presentation, which
necessitated the transfusion of multiple units of blood.
Which of the following tests would most likely lead to the correct diagnosis?
A. Biopsy of the skin lesions
B. Hepatitis serology
C. Measurement of serum antiphospholipid antibodies
D. Measurement of serum anti-neutrophil cytoplasmic antibodies (ANCA)
E. Measurement of serum rheumatoid factor

Rheumatology:Question 52
A 44-year-old man is evaluated because of a 2-year history of “tired feet—pain in his
feet, ankles, and knees after prolonged standing. He knows that he had flat feet as a
youngster, but he had no symptoms until 2 years ago. On physical examination his
gait is normal, and his knees and ankles are brought through full motion without
pain. As he sits on the examining table, his feet look normal.
However, when the patient stands and his feet are viewed from the rear, it is noted
that the arches are collapsed and the ankles and heels turn in.
Which of the following is the best management for this patient?
A. Advise him to reduce the number of hours he is on his feet at work.
B. Suggest that he buy better shoes with more support.
C. Refer him to a podiatrist.
D. Prescribe support stockings.
E. Initiate treatment with a nonsteroidal anti-inflammatory drug.

Rheumatology:Question 53
A 35-year-old man with hemiplegia is referred by the emergency department. He has
antiphospholipid antibody syndrome, which was diagnosed after recurrent episodes
of deep venous thrombosis, and he takes warfarin. Skin examination shows livedo
reticularis. He has a mitral regurgitation murmur. On physical examination, his leg is
normal, without a Homans sign. A CT scan done in the emergency department shows
an infarct in the middle cerebral artery distribution, with no hemorrhage.
Laboratory studies:
Hematocrit 45%
Leukocyte count 7800/μL
Platelet count 110,000/μL
INR 2.1
Results of a peripheral blood smear are pending.
What is the most likely cause of his stroke?
A. Systemic lupus erythematosus with central nervous system vasculitis
B. Thrombotic thrombocytopenic purpura
C. Cerebral embolus
D. Cerebral hemorrhage
E. Patent foramen ovale with paradoxical embolus

Rheumatology:Question 54
A 65-year-old man is evaluated because he has had increasing weakness in the arms
and legs over the past 10 months. He has had gouty attacks in the past that
responded to colchicine and nonsteroidal anti-inflammatory drugs.
On physical examination, he has definite weakness in the hip flexors and quadriceps
muscles, but good strength everywhere else. Patellar reflexes are absent.
Laboratory findings include a serum creatine kinase level of 320 U/L.
Electromyogram shows a mixed pattern of myopathic and neurogenic changes.
Muscle biopsy specimen shows endomysial inflammation and basophilic vacuoles. On
electron microscopy, the vacuoles contain particles resembling myxovirus or
paramyxovirus. Treatment is begun with 80 mg prednisone in divided doses. After 6
weeks, during which time the patients weakness has gradually increased,
methotrexate, 15 mg/week, is added. Four weeks later, the serum creatine kinase
level is 170 U/L, but there is no improvement in muscle strength.
Which of the following is the best next step in this patients management?
A. Add ribavirin.
B. Add intravenous immune globulin.
C. Reduce the prednisone dosage to 20 mg/d.
D. Discontinue the methotrexate and add cyclosporine.
E. Discontinue the methotrexate and taper the dosage of prednisone.

Rheumatology:Question 55
A 72-year-old woman is evaluated because of polymyalgia rheumatica. The diagnosis
was made on the basis of diffuse pain in the neck and shoulder girdle and an
elevated erythrocyte sedimentation rate. She has been treated with prednisone, as
much as 20 mg/d, but the diffuse pain has persisted and progressed to involve her
lower back and buttocks as well as her shoulders and neck. She is able to perform all
activities of daily living but finds all purposeful movement painful.
She is constantly fatigued and reports that her sleep is not restful. She has a history
of migraine headaches that are relieved by large doses of aspirin. She has noted low-
grade fevers in the mornings.
She currently takes no medications; her symptoms were not exacerbated when the
dosage of prednisone was tapered off over a 2-week period.
Review of systems is unremarkable. Her temporal arteries are palpable, firm, and
non-tender. Laboratory studies:
Leukocyte count 12,400/μL
Hemoglobin 8.1 g/dL
Erythrocyte sedimentation rate 98 mm/h
Fasting plasma glucose 143 mg/dL
Serum lactate dehydrogenase 430 U/L
Serum creatinine 0.8 mg/dL
Serum calcium and phosphorus levels are normal, as are results of liver function
tests.
Bilateral temporal artery biopsy specimens reveal atherosclerotic changes but no
vasculitis.
Which of the following is the most appropriate action at this time?
A. Initiate prednisone therapy.
B. Prescribe nortryptyline at bedtime.
C. Prescribe prednisone and methotrexate.
D. Prescribe a short-acting hypnotic agent at bedtime and an aerobic exercise
program
E. Evaluate the elevated erythrocyte sedimentation rate.

Rheumatology:Question 56
A 74-year-old man is hospitalized with diffuse alveolar hemorrhage that developed
abruptly over several days time, starting with small quantities of hemoptysis. His
illness had a prodrome of several weeks, manifested principally by malaise and
arthralgia. Upon admission to the intensive care unit, his serum creatinine level is
7.4 mgldL.
His urinalysis shows 3+ protein, erythrocytes too numerous to count, and occasional
but unequivocal erythrocyte casts. His respiratory status quickly declines to the point
that intubation is necessary. On physical examination, he is noted to have splinter
hemorrhages.
Which of the following autoantibodies is this patient most likely to have?
A. Antineutrophil cytoplasmic autoantibodies
B. Anti-Ro (SS-A) or anti-La (SS-B) antibodies, but not both
C. Anti-Sm antibodies
D. Antiribonucleoprotein antibodies

Rheumatology:Question 57
A 45-year-old man with no history of joint disease is evaluated because of a 6-day
history of aching in his hands, which resolved. The aching was followed by pain in his
wrists, and then pain in his feet as the wrist pain diminished. He has had burning on
urination.
He reports having had pain and redness at the dorsum of the second
metacarpophalangeal joint of his right hand, which hurt a great deal when he would
write. His father, brother, and a paternal uncle have gout.
On physical examination, his temperature is 37.8 °C (100 °F). There is pain with
motion at the right wrist, and he has a small effusion in the right knee. The skin over
the knee is warmer than on the left, and full flexion of the knee elicits pain.
Examination of the penis and prostate reveals no abnormalities.
Arthrocentesis of the right knee yields 10 mL of cloudy fluid. Monosodium urate
crystals are seen by polarized microscopy.
The synovial fluid leukocyte count is 33,000/μL, with 88% neutrophils.
Blood and synovial fluid are sent for culture.

Which of the following is the most appropriate management for this patient?
A. Measure the patient’s serum uric acid.
B. Treat the patient with colchicine.
C. Treat the patient with ceftriaxone.
D. Treat the patient with allopurinol.
E. Inject the right knee with a depot preparation of corticosteroid.

Rheumatology:Question 58
A 23-year-old woman with systemic lupus erythematosus presents with swelling of
the left leg. Her disease was previously manifested by episodes of acute confusion,
nephritis, rash, and mild arthritis; she had a pulmonary thromboembolism 3 years
ago. At that time, she tested highly positive for lupus anticoagulant, and she was
treated with heparin and then warfarin, which she has continued to take. Her
examination is normal apart from the mild leg swelling with some calf tenderness.
Peripheral venous Doppler examinations show evidence of deep femoral venous
obstruction by clot.
Laboratory studies:
Hematocrit 42%
Leukocyte count 5200/μL
Platelet count 120,000/μL
INR 1.9
An MRI is normal, and the measure of IgG anticardiolipin moderately positive. She
asks whether she should stop the anticoagulant therapy.
Which of the following is the next best step in this patient’s management?
A. Substitute aspirin, 80 mg/d, for warfarin.
B. Maintain the present warfarin dosage.
C. Increase the warfarin therapy to achieve an INR of 3.5 for 3 months, then stop
the anticoagulation therapy.
D. Increase the warfarin therapy to achieve an INR 3.5, and maintain it for the long
term.

Rheumatology:Question 59
A 25-year-old woman is evaluated because of a 3-month history of progressive
dyspnea and the development of a dry hacking cough. She has Raynaud’s disease
and polymyositis, which have responded to treatment with prednisone and
methotrexate.
On physical examination, the patient is afebrile, with bibasilar crackles in her chest.
A chest radiograph shows diffuse reticulonodular changes, predominantly in the lung
bases.
Which of the following diagnoses is most likely?
A. Aspiration due to pharyngeal or esophageal dysfunction
B. Interstitial lung disease
C. Pneumonitis due to Pneumocystis carinii
D. Methotrexate-induced lung disease
E. Bronchiolitis with organizing pneumonia
Rheumatology:Question 60
A woman with long-standing rheumatoid arthritis continues to drink more than two
martinis daily. She has not informed her physicians that she does so, even though
she was told not to drink alcohol because of her medication regimen that includes
methotrexate, 15 mg/week. In addition, she has missed getting routine laboratory
tests for assessment of possible toxicity from her medications. On examination, she
has diffuse subepidermal hematomas, evident for the first time, a palpable, slightly
tender liver, and several spider telangiectasias on her abdomen. No ascites is
detected on palpation or percussion. Pitting edema is present to her knees,
bilaterally. Her joint examination is unchanged from previous visits: She has synovial
swelling in both knees, both wrists, and metacarpophalangeal joints.
Laboratory studies:
Hemoglobin 9.6 g/dL
Leukocyte count 3,200/μL
Platelet count 120,000/μL
Erythrocyte sedimentation rate 74 mm/h
C-reactive protein 0.7 mg/dL
Serum alanine aminotransferase 38 U/L
Serum aspartate aminotransferase 42 U/L
Prothrombin time 13s
Serum albumin 2.1 g/dL
Serum globulin 4.6 g/dL
Urinalysis Normal, except for 1+ protein
Which of the following is true about this patients laboratory tests?
A. The low leukocyte count is an indication that she has Felty’s syndrome.
B. A serum haptoglobin determination would be the most helpful test in sorting out
the cause of her mild anemia.
C. The erythrocyte sedimentation rate is not a useful measure of joint inflammation
in this patient.
D. The slight elevations of serum aminotransferase values suggest methotrexate
toxicity as a cause of her apparent liver disease.
E. The elevated serum globulin causes a falsely low C-reactive protein level.

Rheumatology:Question 61
A 65-year-old woman is evaluated because of a 6-month history of malaise, a 4.1-kg
(9-Ib) weight loss, recurrent sinusitis, and a persistent cough. As her history is
obtained, she has some trouble hearing the questions clearly. On physical
examination, she is afebrile. Proptosis is noted, and the mid-portion of the nasal
bridge has a flattened appearance. Both sides of the nasal septum are ulcerated.
Laboratory studies show a hemoglobin of 11.4 g/dL, leukocyte count of 8900/μL and
erythrocyte sedimentation rate of 66 mm/h. She is found to be positive for
rheumatoid factor.
Which of the following tests would be most helpful in determining a diagnosis?
A. Nasal septum biopsy
B. Chest radiograph
C. Measurement of antineutrophil cytoplasmic antibody
D. Sputum culture
E. Measurement of antiglomerular basement membrane antibody

Rheumatology:Question 62
A 38-year-old woman is evaluated because of an 8-month history of aching in many
of her joints, all day and all night. The pain interrupts her sleep several times at
night. In the morning, she does not feel as though she has slept. She also says her
hands and feet swell from time to time.
Approximately 10 months ago, she noted a spreading circular red rash on her thigh.
A physician told her she had Lyme disease and treated her with doxycycline for 3
weeks. Two months later, her joint symptoms and inordinate fatigue began.
Physical examination reveals no abnormalities except for tenderness to pressure
along the thoracic paravertebral muscles and the shoulder girdle muscles. Muscle
strength is 4 to 5 (of 5) throughout. Results of laboratory studies, including serum
creatine kinase levels, are normal, but her erythrocyte sedimentation rate is 33
mm/h. Circulating IgM Borrelia antibodies are not present, but IgG antibodies are.
Which of the following is the best management for this patient?
A. Intravenous ceftriaxone
B. Prednisone
C. An exercise program and amitriptyline at bedtime
D. A nonsteroidal anti-inflammatory drug
E. Muscle biopsy

Rheumatology:Question 63
A 27-year-old woman is evaluated because she has dusky erythema over her palms
and around her eyes. She has also reports difficulty getting up from a reclining
position.
On physical examination, she has an erythematous rash over her face, neck, and
anterior chest that blanches with pressure and clear evidence of proximal muscle
weakness.
Laboratory tests show an erythrocyte sedimentation rate of 45 mm/h, serum
creatine kinase of 450 U/L, and an antinuclear antibody titer of 1:160.
Which of the following antibodies would indicate an increased risk of interstitial lung
disease in this patient?
A. Anti-ribonucleoprotein
B. Anti-histidyl-tRNA synthetase
C. Anti-signal recognition peptide
D. Anticollagen
E. Anti-PM-Sd

Rheumatology:Question 64
A 23-year-old woman with the subacute onset of a malar rash and arthritis of her
hands and wrist is being treated with hydroxychloroquine and celecoxib. Five weeks
after the onset of those symptoms, she is evaluated because of weakness and
difficulty urinating. She has intermittent fever and headaches, as well. Over the past
2 weeks she has noted progressive difficulty starting the urine stream; when it
begins, the quantity voided is small. She has noted no blood, but her urine seems
darker than usual. There has been no dysuria. She is alert, oriented, concerned, thin,
and restless. Her temperature is 38 °C (100.4 °F), pulse rate 84/min, respiratory
rate 14/min, blood pressure 131/80 mm Hg.
Which of the following is the most important to perform at this time?
A. Urinalysis and urine culture
B. Lumbar puncture
C. Evaluation for a distended bladder
D. Careful retinal examination
E. Measurement of total hernolytic complement (CH 50)

Rheumatology:Question 65
An 81-year-old woman is evaluated because of a 1-year history of pain in her right
hip that has increased in severity in the past 2 months. Two years ago, she had total
replacement of the right hip joint for severe osteoarthritis.
On physical examination, she is afebrile. The surface of the hip is warm, and hip
motion increases her pain.
Laboratory studies show a leukocyte count of 6400/μL with a normal differential.
Erythrocyte sedimentation rate is 44 mm/h. Radiograph of the right hip is consistent
with loosening of the femoral component of the prosthesis.
What is the best next step in the management of this patient?
A. Refer her to an orthopedic surgeon for replacement of the loose prosthesis.
B. Refer her to a physical therapist for lower extremity muscle strengthening
exercises.
C. Refer her to an interventional radiologist for arthrocentesis of right hip joint.
D. Treat her with amoxicillin.
E. Order blood cultures.

Rheumatology:Question 66
A 27-year-old woman is evaluated because of subacute onset of symmetrical
proximal muscle weakness with muscle pain. She abuses alcohol and injects cocaine
regularly. She has tested positive for HIV, for which she takes zidovudine. She also
has hypercholesterolemia, for which she takes simvastatin. Her serum creatine
kinase level is 540 U/L. Muscle biopsy specimen shows mitochondrial abnormalities
with depletion of mitochondrial DNA.
Which one of the following is the cause of this patients myositis?
A. Simvastatin
B. Zidovudine
C. Cocaine
D. HIV infection
E. Alcohol

Rheumatology:Question 67
A 24-year-old man has had progressive fatigue, back pain, and stiffness on arising in
the morning, with difficulty putting on his shoes because of back pain. He has lost
6.4 kg (14 Ib). He has taken ibuprofen in low doses without relief. He reports that
his father had back pain.
On physical examination, his temperature is 38.4 °C (101 .1 °F), and his pulse rate
is 108/min. His blood pressure is 122/74mm Hg. There is markedly decreased flexion
of his lumbar spine. There is no evidence of peripheral arthritis or other
abnormalities on examination. Laboratory studies show a leukocyte count of
14,000/μL, hemoglobin of 9.9 g/dL, erythrocyte sedimentation rate of 83 mm/h;
renal and liver function are normal.
Which of the following is the best next step in this patients management?
A. Obtain HLA-B27 analysis from lymphocytes of the patient.
B. Initiate therapy with sulfasalazine and have the patient return in 3 months.
C. Pursue causes other than spondylitis for the elevated erythrocyte sedimentation
rate.
D. Seek a hemolytic cause for this patient’s anemia.
E. Initiate methylprednisolone enemas.

Rheumatology:Question 68
A 28-year-old woman is evaluated because of a 9-day history of arthritis,
temperature to 38.3 °C (101 °F), and chills. These symptoms were followed by pain
and swelling of the second and third metacarpophalangeal and proximal
interphalangeal joints on the right, and the second, third, and fourth
metacarpophalangeal joints on the left.
As swelling of these joints resolved, the right wrist became swollen, warm, and
tender. As the wrist improved, the right knee became swollen, warm, and painful.
The patient developed a mild sore throat. Her last menstrual period ended 6 days
ago. She is in a monogamous sexual relationship.
On physical examination, her temperature is 38.3 °C (101 °F), pulse rate is 100/min,
and blood pressure is 110/70 mm Hg. Her posterior pharynx is red. Her lungs are
clear, and a soft systolic murmur is audible along the left sternal border; her heart
rhythm is regular. There is a 5-mm vesicle surrounded by red skin on the right
forearm. There is exquisite tenderness along the extensor indicis tendon with flexion
of the second metacarpophalangeal joint on the right. An effusion is palpable in the
right knee. Pelvic examination is normal, and there is no cervical discharge.
Which of the following is the best initial treatment for this patient?
A. Codeine sulfate
B. Ibuprofen or indomethacin
C. Ceftriaxone
D. Penicillin
E. Prednisone

Rheumatology:Question 69
Which one of the following factors is predictive of an unfavorable outcome in adults
with polymyositis or dermatomyositis?
A. Presence of a rash
B. The amount of increase in the serum creatine kinase level
C. Presence of dysphagia
D. Age at onset
E. Absence of myositis-specific antibodies

Rheumatology:Question 70
A 34-year-old man is evaluated because of pain and swelling in his hands, wrists,
and ankles for 3 months. The pain makes it difficult for him to open jars or lift
objects. In addition, he has had increasing difficulty climbing stairs and reaching over
his head; this weakness is not associated with pain.
On physical examination, painful swelling is present in the metacarpophalangeal
joints, the proximal interphalangeal joints, wrists, and ankles; grip strength is half
normal. Strength in his knee extensors, neck extensors and flexors, and arm
abductors is 3/5, without pain. Laboratory studies shown a C-reactive protein level of
4.2 mg/dL, hemoglobin of 10.2 g/dL, rheumatoid factor of 1:160, and fluorescent
antinuclear antibody titer of 1:640 (speckled pattern). Urinalysis shows normal
results.
Which of the following is the next best step in this patients management?
A. Order radiographs of the hands and wrists.
B. Measure serum creatine kinase and extractable nuclear antigen.
C. Initiate hydroxychloroquine and ibuprofen therapy.
D. Prescribe several weeks bed rest.
E. Test for hepatitis C and cryoglobulins.

Rheumatology:Question 71
A 68-year-old man is evaluated because of a 2-year history of pain and swelling in
the right knee, which has limited his walking. During the past 6 months he has lost
5.4 kg (12 Ib) and he has not been his usual robust, energetic self. He has continued
to work long hours. He has not had any lower extremity injury.
One year ago, fluid was removed from the swollen joint. His serum uric acid level
was 9.4 mg/dL; he was told he had gout and was treated with colchicine, 0.6 mg/d,
which he took for only 3 days. The knee pain and swelling waxed and waned, but
never disappeared completely. Being active in his work, he did not think much of the
problem and has come today for evaluation only because his wife insisted on it.
He was in Korea in the Army in 1952. A tuberculin skin test then was positive, but
chest radiographs then and every 2 years since then have been normal. He takes
nonsteroidal anti-inflammatory drugs that do reduce the swelling transiently.
No abnormalities are noted on physical examination except for palpable fluid in the
right knee. There is also periarticular swelling, and knee flexion and extension are
reduced by 20 degrees. Aspirated synovial fluid shows leukocyte count of 16,000/μL,
with 66% neutrophils and 34% mononuclear cells. No crystals are present. Results of
Gram stain and stain for acid-fast bacilli are negative. Subsequent routine cultures
are negative. His peripheral blood leukocyte count is normal. Erythrocyte
sedimentation rate is 54 mm/h. Serologic test for rheumatoid factor is positive.
Radiograph of the knee shows small, well-delineated subchondral bone erosion on
the non-weight-bearing surface of the distal femur. MRI shows no damage to other
structures of knee.
Which of the following is the best next step in this patients management?
A. Bone scan to document osteonecrosis
B. Arthroscopy to wash out loose bodies
C. Synovial biopsy to stain and culture tissue for acid-fast bacilli
D. CT scan to document osteonecrosis

Rheumatology:Question 72
A 28-year-old woman with systemic lupus erythematosus diagnosed 4 months ago
has been treated for 3 months with hydroxychloroquine, 200 mg twice daily. The
symptoms that led to the diagnosis included polyarthritis, hair loss, fatigue, and
ulcers on her palate.
The patient had a complete eye examination before treatment with
hydroxychloroquine was begun. The ophthalmologist approved of treatment and told
her she was “near-sighted” and recommended she wear glasses for driving. She
obtained the glasses and noted that her vision was much improved. However, she
has noted some blurring of vision the past 3 weeks. Her fiancé, a physician, told her
to stop the hydroxychloroquine and never take it again.
Which of the following is true in reference to this patient?
A. She should discontinue hydroxychloroquine therapy.
B. Hydroxychloroquine does not cause ocular toxicity.
C. Blurred vision may occur early in the course of treatment with
hydroxychloroquine.
D. Myopia is an adverse event of hydroxychloroquine therapy.

Rheumatology:Question 73
A 68-year-old man is evaluated because of increasing pain in the second and third
metacarpophalangeal joints of both hands. He has had osteoarthritis for many years
and has had arthroplasty/joint replacement of both shoulders, the left knee, and
right ankle. Other comorbidities include insulin-dependent diabetes mellitus,
intractable erectile dysfunction, and slowly progressive congestive heart failure.
On physical examination he is found to have hard exostoses around all
metacarpophalangeal joints, without soft tissue swelling, and limited motion of the
joints on which he had surgery. Abnormalities include bilateral cataracts,
tachycardia, intermittent third heart sound, and a slightly enlarged liver without
hepatojugular reflux.
Which one of the following tests should be performed?
A. Serum iron and iron-binding capacity
B. Serum rheumatoid factor and fluorescent antinuclear antibody
C. Radiographs of both shoulders
D. Thallium stress test
E. Serum free testosterone and thyroid function tests

Rheumatology:Question 74
A 44-year-old married woman and the mother of four children ages 3 through 8
years, is evaluated because of a 2-week history of aching in the joints of her wrists,
hands, and knees. Pain and swelling were severe for approximately a week, then
subsided to aching. The pain is worse in the morning and abates somewhat with
activity. On physical examination, there is tenderness with pressure over the dorsa
of the wrists and pain with wrist motion. One side of the patients face shows faint
redness. She has noticed patchy sloughing of the epidermis of the hands.
What is the best means to diagnose this patient’s illness?
A. Test for antinuclear antibody
B. Test for circulating rheumatoid factor
C. Test for circulating IgM anti-parvovirus B19 antibodies
D. Radiographs of the wrists
E. Therapeutic trial of prednisone, 10 mg/d for 7 days

Rheumatology:Question 75
A 58-year-old woman is evaluated because of a i-year historyof worsening pain in
her right knee. Itis a deep, diffuse ache, not associated with swelling or tenderness.
It is worse at the end of the day. The
patient is relatively free of pain when she awakens in the morning, but says that she
never has a good nights sleep. She has no morning stiffness. Other problems include
an irritable bowel, frequent migraine headaches, and diffuse aching in her shoulders
and neck. On physical examination, her knee is normal. She has tender points in the
trapezii, occipital areas, and over the lateral elbow epicondyles bilaterally. She is
obese. All results of routine laboratory tests are normal.
Which of the following is the most appropriate next step in this patient’s
management?
A. Prescribe celecoxib for pain
B. Order bilateral hip radiographs
C. Prescribe gradual isometric exercise for both knees
D. Refer her to a podiatrist for cushioned shoes
E. Inject the right knee with triamcinolone hexacetonide

Rheumatology:Question 76
A 39-year-old woman with a 3-year history of rheumatoid arthritis is evaluated
because of slight shortness of breath of new onset when she climbs the stairs in her
home. She also has a dry hacking cough of 3 weeks duration. She has been active,
working part-time as a secretary as well as caring for two young children at home.
She has been taking methotrexate, 20 mg by mouth each week, and folic acid, 1
mg/d, for 1 year, and she says her joints “have been great.”
On physical examination, her temperature is 37.7 °C (99.8 °F). Fine crackles are
audible at the bases of both lungs on deep inspiration. There is some tenderness
with full palmar flexion and dorsiflexion of the right wrist; the rest of the joints are
brought through full range of painless motion, and there is no other evidence of
active synovitis. Serum alanine aminotransferase level is 44 U/L and serum aspartate
aminotransferase level is 50 U/L. Complete blood count and serum albumin levels
are normal. Chest radiograph is normal.
Which of the following is the best next step in this patients management?
A. Order liver biopsy.
B. Obtain radiographs of wrists.
C. Stop the methotrexate.
D. Initiate prednisone therapy.

Rheumatology:Question 77
A 54-year-old woman is evaluated because of severe pain in her left calf of sudden
onset. She has had seropositive rheumatoid arthritis for 13 years involving her
hands, wrists, and knees. Her knees have been injected four or five times over this
period with triamcinolone hexacetonide for pain and large effusions. She has
responded well in the past 2 years to a combination of etanercept and methotrexate.
Pain in her knees has diminished, but the effusions have persisted. She is able to
shop for several hours and to take easy hikes in the foothills with her grandchildren.
On physical examination, there is a moderate effusion in the right knee. The left
knee has a small amount of fluid present, and her left calf is tender to mild
compression and is 5 cm larger in diameter than the right calf. Her temperature is
37.6 °C (99.6 °F) and her pulse is regular at 84/min; she is not dyspneic.
Which of the following would be contraindicated at this time?
A. Administration of enoxaparin
B. Aspiration of the fluid in the left knee for cell count and bacterial culture
C. Administration of hydrocodone bitartrate with acetaminophen
D. Intravenous injection of methylprednisolone
E. Intra-articular injection of triamcinolone hexacetonide after aspiration and culture
of the fluid in the left knee
ANSWERS
Rheumatology:Question 1
The correct answer is C
Educational Objectives
Evaluate cardiovascular risk factors in a patient with systemic lupus erythematosus.
Critique
Accelerated atherosclerosis is a major cause of death in systemic lupus
erythematosus, and this patients elevated blood pressure, hypercholesterolemia, and
history of cigarette smoking put her at increased risk. She must be treated for her
systemic lupus erythematosus, but also these risk factors must be addressed,
particularly the glucose level, in light of her long-term corticosteroid therapy.
The physician should aggressively recommend smoking cessation and weight loss,
and prescribe drug therapy to achieve a blood pressure below 130/80 mm Hg and
reduce her low-density lipoprotein cholesterol to less than 100 mg/dL.

Rheumatology:Question 2
The correct answer is B
Educational Objectives
Diagnose an idiosyncratic reaction to methotrexate therapy.
Critique
Although the mechanism is not known, patients with rheumatoid arthritis who are
treated with methotrexate occasionally have nodules that increase in size and in
number. The nodules are not related to efficacy of the drug, and it is rare that
methotrexate treatment would need to be stopped because of them.
Biopsy of a new nodule would show typical rheumatoid granulomas that can be easily
distinguished from those of sarcoid nodules or tuberculosis. Tendinous xanthomas
and tophi are part of a differential diagnosis of subcutaneous nodules; xanthomas
often have a characteristic orange cast and do not appear in a short period of time.
Low doses of salicylates (<500 mg/d) may increase serum urate levels slightly by
inhibiting tubular secretion of urate, whereas high doses such as those taken by this
patient inhibit both tubular reabsorption and secretion with a net effect of decreasing
serum urate concentrations. In addition, there is some evidence that high levels of
serum uric acid are anti-inflammatory, a possible reason that there are many fewer
individuals than would be expected who have both gout and rheumatoid arthritis.

Rheumatology:Question 3
The correct answer is E
Educational Objectives
Recognize the clinical presentation of the tumor necrosis factor receptor-associated
periodic syndrome.
Critique
This patient presents with several of the classic findings of the tumor necrosis factor
receptor-associated periodic syndrome (TRAPS). The diagnosis is established by
sequencing TNFRSF1A. His current episode, with fever and a migratory erythematous
rash overlying an area of myalgia, and a normal creatine kinase level, is quite
characteristic of TRAPS and is not seen in any of the other hereditary periodic fever
syndromes.
In familial Mediterranean fever, the rash is most typically on the lower leg, ankle, or
dorsum of the foot (erysipeloid erythema). In hyperimmunoglobulinemia D with
periodic fever syndrome (HIDS), the rash is often a more diffuse maculopapular
eruption. In the cryopyrin-associated syndromes, the rash is urticarial.
Also consistent with TRAPS are the duration of the episodes (>1 week) and the
history of therapeutic response to corticosteroids. Adult patients with TRAPS
sometimes give a history of having been diagnosed with juvenile rheumatoid
arthritis.
Episodes of pleurisy, abdominal pain, and arthritis can also be seen in familial
Mediterranean fever, but would typically last 1 to 3 days. The history of conjunctivitis
in the mother is also consistent with TRAPS.
A trial of colchicine or a genetic search for pyrin mutations could establish the
diagnosis of familial Mediterranean fever, were this a more plausible option. The
patient’s Arab ancestry raises the possibility of familial Mediterranean fever, but, as
noted above, the other clinical features are not consistent with the diagnosis. In
addition, TRAPS has been reported in Arab patients with periodic fever syndromes.
MRI of the calf might be ordered on the suspicion of dermatomyositis, but the
migratory myalgia and rash described in this case are atypical of the inflammatory
myopathies, and MRI would not distinguish these conditions from TRAPS.
The serum ferritin level is relevant to the diagnosis of adult-onset Still’s disease.
Clinical features against this diagnosis include the fever pattern (continuous fever in
this case, quotidian in Still’s disease) and the nature of the skin rash. Serum ferritin
levels, when measured, are elevated in patients with TRAPS as part of the acute-
phase response and would therefore not necessarily distinguish adult-onset Still’s
disease from TRAPS.

Rheumatology:Question 4
The correct answer is A
Educational Objectives
Distinguish inflammatory from noninflammatory back pain
Critique
Although there are no localizing neurologic findings on examination, this patient has
back pain that is worse with coughing and sneezing and is of a clear-cut unilateral,
radicular nature. These symptoms are most consistent with a small herniated disk.
It is unlikely that a patient as young as 30 years will have significant radiographic
changes of osteoarthritis, such as those described in option B. Furthermore, although
osteoarthritis can cause unilateral back pain with posterior leg pain, particularly in
the setting of facet arthropathy, pain from facet arthropathy does not worsen with
increasing intra-abdominal pressure, such as is induced by coughing and sneezing,
and it does not radiate below the knee.
The radiographic changes described in option C are those of sacroiliitis, one of the
major causes of inflammatory back pain (the other being spondylitis). Although the
skin rash is most likely caused by psoriasis, which is associated with sacroiliitis and
spondylitis, the patient has none of the symptomatic or laboratory features
suggestive of an inflammatory source of back pain. Specifically, his pain is not worse
in the morning and does not improve with activity, and the markers of inflammation
often found in spondyloarthropathy (elevated erythrocyte sedimentation rate and C-
reactive protein concentration) are normal.
Although patients with advanced ankylosing spondylitis have recently been
recognized to have an increased incidence of osteoporosis, which can lead to
compression fractures (manifested by anterior wedging of an affected vertebral
body), it would be distinctly unusual for a man of his age, even with ankylosing
spondylitis, to develop back pain on this basis in the absence of significant trauma.
Furthermore, back pain from a compression fracture is acute, severe, self-limited,
and generally nonradiating.

Rheumatology:Question 5
The correct answer is C
Educational Objectives
Diagnose Takayasu’s arteritis.
Critique
This young Asian woman has a classic presentation of Takayasus arteritis. Her illness
is in the inflammatory phase, in which constitutional symptoms, normochromic
anemia, thrombocytosis, and elevation of acute-phase protein levels are evident.
Bruits over the large arteries are likely to be heard on physical examination; the
vessels most commonly involved are the subclavian, axillary, carotid, renal, and
femoral arteries, in addition to the aorta itself. Her back pain is probably due to
inflammation of the thoracic aorta, and the achiness in her arms is actually
intermittent claudication caused by narrowing of the subclavian artery.
A magnetic resonance angiogram is useful in confirming the diagnosis; in Takayasus
arteritis it demonstrates thickening and edema of the walls of large vessels and long,
smooth, tapering of vascular lumina. Conventional aortography is probably required,
as well, to gauge her central aortic pressure accurately, because the subclavian
stenoses prevent reliable blood pressure measurements in the arms.
Hospitalization, blood studies, intravenous saline, and administration of iron are
incorrect management procedures because they suggest diagnostic testing or
therapeutic interventions without any further data from a careful physical
examination.

Rheumatology:Question 6
The correct answer is C
Educational Objectives
Recognize the joint involvement and the temporal pattern of pain in osteoarthritis.
Critique
This patient has osteoarthritis, in which the pain is due to bone marrow edema, to
pressure on bone denuded of cartilage, and to altered gait mechanics. Pain increases
with activity and weight bearing in patients with osteoarthritis.
In patients with inflammatory arthritis (rheumatoid arthritis, psoriatic arthritis,
spondylitis) morning stiffness usually lasts more than 30 minutes and improves with
activity. Although psoriatic arthritis involves distal interphalangeal joints, usually
active joints are red, warm, and tender. Early in development of distal
interphalangeal nodes seen in osteoarthritis, the nodes may be red and soft; the
inflammation usually resolves spontaneously. If patients have distal interphalangeal
inflammation and back pain or atypical patterns of arthritis, it is important to ask
about a family history of psoriasis or inflammatory bowel disease.
The patient should be referred to a physical therapist for formal instruction in muscle
strengthening/joint protection exercises. This patient has no factors such as
alcoholism or corticosteroid therapy that would predispose her to osteonecrosis.
Rheumatology:Question 7
The correct answer is E
Educational Objectives
Recognize the dangers of gout therapy in patients with renal failure, and the
usefulness of oral corticosteroids in the treatment of acute gouty arthritis.
Critique
Oral prednisone is the safest agent for this patient and is very effective in
suppressing acute gouty inflammation, even though it may transiently further
exacerbate his hyperglycemia. Reasonable therapy for this patient would be
prednisone, 30 mg orally for 2 days, followed by a rapid tapering of the dosage.
Oral colchicine would be inappropriate; there are few patients angrier than those
with acute podagra and colchicine-induced diarrhea. Hypouricemic therapy with
allopurinol would be appropriate for this patient with elevated uric acid and frequent
gouty attacks, but only after the acute attack has subsided. Indomethacin could
worsen this elderly patient’s mild renal failure. Intravenous colchicine should never
be administered to patients with abnormal renal function because of possible bone
marrow suppression.

Rheumatology:Question 8
The correct answer is D
Educational Objectives
Diagnose C1-C2 subluxation in a patient with active, severe rheumatoid arthritis and
neurologic symptoms.
Critique
The shooting pains in her arms, hyperreflexia, and hypertonia suggest that this
patient may have atlantoaxial subluxation (AAS) secondary to erosion or stretching
of the transverse ligament that holds the odontoid process in place anterior to the
spinal cord. Impingement or compression of the cervical cord can result, especially
when the neck is flexed forward. Although this woman would probably not be a
candidate for spinal fusion/fixation, it is essential to document the presence or
absence of subluxation, particularly when general anesthesia is planned for revision
of her arthroplasty.
The mortality rate of patients with AAS is eight times higher than that of unaffected
patients. The magnitude of AAS can appear smaller when measured by functional
MRI rather than by functional radiography; for this reason, MRI is the modality of
choice for detecting possible spinal cord compression in severe rheumatoid arthritis.
Although the combination of alopecia and oral ulcers may suggest that the patient
also has systemic lupus erythematosus, her regimen of methotrexate is more than
sufficient reason to explain their presence, and autoantibody testing is not a high
priority. Digital infarcts are among the most common forms of vasculitis in
seropositive rheumatoid arthritis. They can mimic the lesions in cryoglobulinemia. In
the absence of hepatitis C infection, cryoglobulins are rarely present in rheumatoid
arthritis. Addition of a tumor necrosis factor-α (TNF-α) inhibitor may be appropriate if
serious cervical spine disease does not preclude revision of her hip arthroplasty. In
this case it would be appropriate to defer TNF-α inhibition therapy until she is stable
after surgery. It is also important to ensure that there are no contraindications to
anti-TN F-ct therapy, such as evidence of infection or previous tuberculosis.
This woman with a chronic inflammatory disease who has been limited to bed and
chair for many years is probably osteoporotic. If successful surgery allows her to
begin walking in rehabilitation therapy, bone protection is essential to minimize the
risk of stress fracture or fracture during falls.
Rheumatology:Question 9
The correct answer is C
Educational Objectives
Distinguish between radiographic findings in patients with “arthritis robustus” and
the more common variety of rheumatoid arthritis.
Critique
This man is one of a subset of patients with rheumatoid arthritis frequently called
arthritis robustus; it is common in laborers who use their hands. Patients with this
disease have erosive and destructive rheumatoid arthritis but very little pain. Their
strength remains high and they have minimal subluxation of joints. Radiographs of
the hands would likely show metacarpophalangeal erosions.
This man’s cough is probably related to the angiotensin-converting enzyme inhibitor
that he began to take for his hypertension. Many patients who take these drugs
develop a persistent, annoying, dry cough that is not associated with abnormal
pulmonary function tests or abnormal chest radiographs. If he were taking
methotrexate, pulmonary toxicity from this drug, which can cause alveolitis, would
be a consideration. The most common pulmonary complication of rheumatoid
arthritis is pleuritis with an associated pleuritic chest pain. Interstitial fibrosis is
accompanied by decrease in diffusing capacity and, when progressive, by dyspnea,
but not usually with a dry cough. Because he has subcutaneous nodules at the
elbows consistent with rheumatoid nodules, there is a high probability that his
rheumatoid factor test would be positive. It is unlikely that he would have a
neutropenia; he does not have an enlarged spleen that would suggest the possibility
of Felty’s syndrome. His medications are not those that are associated with
neutropenia. Proteinuria in rheumatoid arthritis can occur from secondary
amyloidosis, but he had no ankle edema; furthermore, amyloidosis is very rare in
rheumatoid arthritis. The absence of cellular elements in the urine suggest that the
proteinuria is not related to tubular disease from aspirin.

Rheumatology:Question 10
The correct answer is C
Educational Objectives
Diagnose and treat familial Mediterranean fever.
Critique
This patient has familial Mediterranean fever (FM F), and the treatment of choice is
prophylactic daily oral colchicine.
Clinical features favoring the diagnosis of FMF include the intermittent nature of the
illness, with periods of well-being between the attacks, the duration of the episodic
abdominal pain, documentation of inflammation during an episode, normal
gynecologic and gastrointestinal workup, and the history of an appendectomy as a
child with the finding of a “normal appendix” (patients with FMF sometimes show
evidence of serosal inflammation, but not the inflammation of the mucosal surface
usually seen with an acute appendicitis).
Fluctuation of symptoms with the menstrual cycle and relief on oral contraceptives
have been well documented in both FMF and tumor necrosis factor receptor-
associated periodic syndrome (TRAPS). Although Ashkenazi Jews and Italians were
not initially thought to be at high risk for FMF before the advent of genetic testing,
these two ethnic groups accounted for almost half of mutation-positive American
patients in a recent National Institutes of Health series.
The history of “pleurisy” in the paternal uncle further raises the suspicion of FMF, and
the documentation of two mutations (which happen to be very common in the Italian
and Ashkenazi Jewish populations) confirms the clinical diagnosis. Between 10% and
15% of FMF patients, particularly patients with symptoms that are more severe than
those of this patient, have serum IgD levels above the upper limit of normal,
although the elevations are usually modest. An increased fibrinogen level can be
seen in any of the hereditary periodic fever syndromes.
Oral contraceptives were occasionally used in women patients with FMF before the
advent of colchicine prophylaxis, but contraceptives are generally less effective than
colchicine in preventing attacks and have not been documented to reduce the risk of
amyloidosis. Prednisone is generally not effective in aborting the attacks of FMF.
Etanercept is currently investigational in the small number of patients with FMF who
do not respond to colchicine, but is a much more expensive alternative. Interferon
alfa has also been tried in colchicine nonresponders, and is sometimes effective, but
it has the disadvantages of being injectable, of needing to be given as early as
possible into an attack, and of causing low-grade fever and flu-like symptoms.

Rheumatology:Question 11
The correct answer is D
Educational Objectives
Distinguish between spondyloarthropathy and other common causes of inflammatory
arthritis.
Critique
The patient likely has a history of bloody diarrhea. Her history and physical findings
are consistent with inflammatory arthritis. The pattern of involvement (oligoarticular,
asymmetric, predominantly of the lower extremities) is characteristic of the
spondyloarthropathies, particularly those due to an enteropathic process. In addition,
dactylitis (generalized swelling of an entire digit, usually involving the toes) is
commonly seen in the spondyloarthropathies. The subsequent development of
spondyloarthropathy is associated with enteric infection with certain pathogens, as
well as ulcerative colitis and Crohn’s disease. Bloody diarrhea is suggestive of one of
these disorders; more information about the duration and nature of the bloody
diarrhea would help to distinguish among these entities.
Although rheumatoid arthritis is the most commonly diagnosed rheumatic disorder in
young women, the pattern of joint involvement and the presence of dactylitis in this
patient are not typical of this disease. Polyarticular symmetric arthritis, particularly of
the proximal small joints of the hands and wrists, is most common in rheumatoid
arthritis. Patients with spondyloarthropathy are seronegative for both rheumatoid
factor and antinuclear antibody.
This patients history and physical examination are not suggestive of gouty arthritis,
which, in the absence of renal failure, virtually never occurs in premenopausal
women. Although patients with psoriatic arthritis may have elevated uric acid levels
due to the rapid cell turnover associated with psoriasis, this patient has no psoriatic
skin lesions. The presentation of gout is usually an acute, intense, highly
inflammatory arthritis. Although it can involve the knees, ankles, or toes (often
asymmetrically), the shoulder joint is rarely, if ever, inflamed as a result of gout.

Rheumatology:Question 12
The correct answer is E
Educational Objectives
Diagnose and empirically treat giant cell arteritis.
Critique
This patient with headaches and diplopia almost certainly has giant cell arteritis.
Weight loss is also symptomatic of this illness, but it is not a classic feature. Among
patients with giant cell arteritis, 75% note headaches of new onset. Diplopia,
although it is present in only approximately 10% of patients with giant cell arteritis,
has a high positive predictive value for a diagnostic temporal artery biopsy.
When the diagnosis of giant cell arteritis is strongly suspected, as it should be in this
patient, treatment with corticosteroids should be started immediately—even before it
is possible to obtain a temporal artery biopsy. Although most patients with giant cell
arteritis have an elevated erythrocyte sedimentation rate, approximately 11% of
patients may have an erythrocyte sedimentation rate <50 mm/h. Therefore, the test
should not be the criterion to decide which patients should undergo temporal artery
biopsy. Migraine headaches of new onset would be extremely uncommon in a 78-
year-old woman. Moreover, the duration of migraines is typically far shorter than 8
days, and diplopia would be a very unusual feature for a migraine headache
syndrome.

Rheumatology:Question 13
The correct answer is C
Educational Objectives
Recognize hemochromatosis as a cause of osteoarthritis in a relatively young person.

Critique
Because metacarpophalangeal joint involvement is uncommon in sporadic
osteoarthritis, especially in a fairly young man, the differential diagnosis broadens to
other entities: in this case, hemochromatosis. Radiographs of the hands show hook-
like osteophytes directed proximally at the third and fourth metacarpal heads on the
right, findings typical of hemochromatosis arthropathy. The best screen for
hemochromatosis is to measure serum iron and iron-binding capacity. Arthritis is the
most common extrahepatic finding and may be the sole clinical manifestation of
hemochromatosis.
Hip joint arthritis associated with ankylosing spondylitis is more common in children
and is inflammatory rather than presenting as osteoarthritis. HLA-B27 testing is
useful on occasion (although 8% of white men in the United States test positive for
HLA-B27) if ankylosing spondylitis is suspected. There is little reason to suspect
gout, which is usually monoarticular. Although involvement of small joints of the
hands is common in rheumatoid arthritis, the diagnosis of rheumatoid arthritis is
made on clinical grounds, not with laboratory tests. Patients with hemochromatosis
can develop diabetes mellitus, but the iron studies are more specific than fasting
blood glucose testing would be in this case.

Rheumatology:Question 14
The correct answer is E
Educational Objectives
Recognize the diagnostic criteria for antiphospholipid antibody syndrome.
Critique
No treatment is indicated. This patient does not meet classification criteria for
antiphospholipid antibody syndrome because she has had only one first-trimester
loss. Pregnancy-related criteria include one or more unexplained deaths of
morphologically normal fetuses at or after the 10th week of gestation; or one or
more premature births of morphologically normal neonates at or before the 34th
week of gestation; or three or more unexplained consecutive spontaneous abortions
before the 10th week of gestation. Therefore, no treatment is necessary.
If she did have antiphospholipid antibody syndrome and recurrent pregnancy loss,
the preferred treatment would include heparin and aspirin. Both unfractionated
heparin and low-molecular-weight heparin have been used successfully.
Therapeutic doses of heparin are necessary only if the woman has a history of
thrombosis. Warfarin is not appropriate during pregnancy because of its teratogenic
potential.

Rheumatology:Question 15
The correct answer is D
Educational Objectives
Diagnose colchicine myopathy.
Critique
As many as 12% of transplant recipients treated with cyclosporine develop gouty
arthritis. Prophylactic colchicine is appropriate before starting allopurinol treatment.
However, of the drugs this patient is taking, coichicine is the most likely cause of his
symptoms and the therapy should be discontinued. After 4 weeks of colchicine
therapy, he has developed a myopathy.
Cyclosporine, allopurinol, and mycophenolate mofetil are not often associated with
myopathy. There is no reason to suspect that the patient has developed an
inflammatory polymyositis, which would require treatment with high doses of
prednisone.

Rheumatology:Question 16
The correct answer is C
Educational Objectives
Prescribe inhibitors of tumor necrosis factor-α in spondyloarthropathies refractory to
standard therapy.
Critique
When patients with ankylosing spondylitis become fixed in forward flexion of their
dorsal and cervical spines so that they no longer can see the horizon, their activity
levels and quality of life are enormously compromised. The U.S. Food and Drug
Administration has approved use of tumor necrosis factor-α inhibitors in active
ankylosing spondylitis, and this patient is a good candidate for one of these drugs.
The result of the power Doppler sonography (PDS) confirms the activity of his
disease; this technique gives evidence of inflammation at the entheses that is equal
to if not better than that obtained by MRI. Therefore, MRI in this patient would be
redundant. PDS has been of proved benefit in determining the depth and extent of
erosions in rheumatoid arthritis.
Although spinal exercise to diminish the rate of fixed forward flexion of the dorsal
and cervical spines is appropriate, it should be started early in the disease, and there
are no data to suggest that forced stretching of the cervical spine in active disease
can significantly retard progressive fixed flexion.
When added to methotrexate in some cases of rheumatoid arthritis, cyclosporine has
been a useful adjunct therapy. There are, however, no data to support its use in
ankylosing spondylitis. Prednisone, 5 mgld, would not appreciably retard progression
of the disease. Indeed, spondylitis with active enthesopathy is remarkably refractory
to doses of prednisone that can suppress synovitis in rheumatoid arthritis.

Rheumatology:Question 17
The correct answer is C
Educational Objectives
Order joint fluid analysis and culture to rule out infection in patients with rheumatoid
arthritis who have had total joint replacement.
Critique
Joints that have been replaced with prostheses rarely are involved in flares of
rheumatoid arthritis but do have a predisposition for infection. If there is any
suggestion of fluid in or around such a joint, it must be aspirated as soon as possible
and sent for culture, Gram stain, and cell count.
This woman has several factors that have predisposed her to develop an infection in
her prosthetic joint: an ulcer on the bottom of her foot, therapy for arthritis that
inhibits normal responses to bacteria and mycobacteria, and diabetes mellitus.
Although patients taking tumor necrosis factor-α inhibitors are at an enhanced risk
for reactivation of tuberculosis, the most probable cause of her right knee pain and
swelling is an infection with gram-positive bacteria. Eventually, a bone scan to
assess the possibility of coexisting osteomyelitis will be needed, but the diagnostic
challenge at this time is to diagnose sepsis or eliminate it as a possibility.

Rheumatology:Question 18
The correct answer is E
Educational Objectives
Diagnose amyloidosis in a patient with a hereditary periodic fever syndrome.
Critique
On the basis of this patient’s clinical history, the most likely diagnosis is the Muckle-
Wells syndrome (MWS) complicated by systemic AA amyloidosis. Rectal biopsy is a
relatively noninvasive way of testing for AA amyloidosis, and, if positive, would
demonstrate apple-green birefringent material when stained with Congo red and
viewed under polarized light. The sensitivity of rectal biopsy in AA amyloidosis
ranges from 75% to 85%.
Among the features supporting the diagnosis of MWS are the history of arthritis,
high-frequency sensorineural hearing loss, and fever with limb pain. Also supportive
of the diagnosis are the urticaria-like rash and what appears to be a dominant mode
of inheritance. Digital clubbing and pes cavus are sometimes seen in MWS.
Laboratory findings, including proteinuria, hypoalbuminemia, hypercholesterolemia,
and a noncellular urine sediment, are consistent with the nephrotic syndrome; the
normal fasting plasma glucose level excludes the possibility of diabetic nephropathy.
Elevations in the sedimentation rate could be due either to the inflammatory state of
MWS or to the nephrotic syndrome.
Approximately 25% of patients with MWS develop systemic amyloidosis due to the
deposition of SAA, an acute-phase reactant. Deposits may be found in the kidneys,
liver, gastrointestinal tract, thyroid, adrenal glands, and testes.
A test for a cysteine mutation inTNFRSF1A would be positive in a patient with tumor
necrosis factor receptor-associated periodic syndrome (TRAPS) rather than MWS.
Deposits of β-microglobulin in the joints occur in the form of amyloidosis associated
with hemodialysis. Urine mevalonic acid levels are increased during the febrile
attacks of the hyperimmunoglobulinemia D with periodic fever syndrome (HIDS),
which has not been associated with systemic amyloidosis. AA amyloidosis generally
does not cause neuropathy, and would not cause neuronal light chain deposits.

Rheumatology:Question 19
The correct answer is B
Educational Objectives
Recall the pathogens associated with reactive arthritis and the distinction between
likely clinical courses based on infecting organisms.
Critique
The diagnosis in this patient is reactive arthritis; she developed oligoarticular
inflammatory arthritis after a presumed enteric infection. More than half of patients
with reactive arthritis are asymptomatic after 4 to 6 months.
Reactive arthritis has been associated withShigella flexneri, several Salmonella
species, Campylobacter, Yersinia enterocolitica, and Clostridium difficile but has not
been reported after enteric infection with Escherichia coli. Treatment of the
underlying bacterial infection has not been shown to affect the incidence of
postinfectious reactive arthritis.
There is no distinction between the clinical courses of patients with reactive arthritis
following various enteric infections. Patients who develop reactive arthritis
afterChlamydia urinary tract infections, however, are more likely to have chronic or
recurrent disease. Acute arthritis is seen in disseminated Neisseria gonorrhoeae
infection. It is not associated with the development of subacute or chronic reactive
arthritis but rather acute, often migratory arthritis on the basis of immune complex
disease or true articular infection (septic arthritis). Arthritis associated with
gonorrhea is effectively cured by appropriate antibiotic treatment.

Rheumatology:Question 20
The correct answer is D
Educational Objectives
Recognize the cardinal features of Cogan’s syndrome.
Critique
The patient’s vertigo and hearing loss suggest an inner ear problem. Her
presentation now with ocular inflammation suggests the possibility of an “eye-ear”
syndrome and should raise suspicion of Cogan’s syndrome, which is the combination
of sensorineural hearing loss (often accompanied by vertigo) and eye inflammation.
Virtually any form of eye inflammation can occur in Cogan’s syndrome (such as
scleritis, episcleritis, uveitis), but the classic eye finding is nonsyphilitic interstitial
keratitis, which manifests itself in a manner identical to this patients symptoms.
A diagnosis of Menieres disease is incorrect because it would not account for the eye
symptoms as well as the ear problem. An acoustic neuroma might cause vertigo and
hearing loss, but is unlikely to be bilateral and would not explain the ocular findings.
A diagnosis of viral conjunctivitis does not explain the entire clinical picture. Vertigo
and scleritis are not features of giant cell arteritis.

Rheumatology:Question 21
The correct answer is C
Educational Objectives
Diagnose trochanteric bursitis.
Critique
Many patients with pain over the greater trochanter describe it as hip pain. Often
patients can point with one finger to the source of the pain. Actively resisted
abduction of the hip worsens the pain. The treatment of choice is a corticosteroid
injection. The physician marks the point of pain, places the needle perpendicular to
the skin, pointed toward the painful area, and the corticosteroid (and local
anesthetic) is deposited in the trochanteric bursa. Unless patients are slender, it is
usually necessary to use a spinal needle. In ideal circumstances, when the tip of the
needle touches the inflamed bursa, the patient will complain of increased pain.
Trochanteric bursitis cannot be demonstrated on radiograph of the hip. Formal
instruction in exercises to stretch the iliotibial band and strengthen the gluteus
medius and minimus muscles may be helpful, as will nonsteroidal anti-inflammatory
drugs or hot packs. They might reduce symptoms transiently, but the pain will recur.

If the problem is not corrected, or if it recurs, then other causes of lateral hip pain
must be considered. These include hip disease, spinal disease, and compressive
neuropathy of the subcostal nerve (especially in patients with scoliosis) or
iliohypogastric nerve (pain reproduced with digital pressure at the superior crest). A
tear of the gluteus medius tendon must also be ruled out (if the patient stands on
the affected leg and the pelvis tilts, suspect this diagnosis and do an MRI to
document). If all else fails, bursectomy can be considered. The physical examination
indicates that the source of pain is not the hip joint itself; therefore, a radiograph of
the hip is not warranted at this time.

Rheumatology:Question 22
The correct answer is A
Educational Objectives
Recognize transverse myelitis, a rheumatologic emergency, and choose the
appropriate diagnostic study.
Critique
The sudden onset of a neurologic deficit in the lower extremities with associated
signs of bladder or bowel dysfunction is a medical emergency. The most likely
diagnosis is transverse myelitis, a condition that occurs with some frequency in
patients who have systemic lupus erythematosus. An MRI is the most sensitive test
for detecting spinal cord edema secondary to ischemia.
The normal flexion and absence of tenderness in the bones of the spine suggest an
intramedullary lesion; therefore, it is inappropriate to examine the structures
surrounding the spinal cord by radiograph or CT. Although the results of
cerebrospinal fluid analysis may be abnormal, MRI should be done to define the
extent, location, and morphology of the lesion. Antiphospholipid antibodies are often
found in transverse myelitis, but testing for them is not the procedure of choice for
diagnosing transverse myelitis.

Rheumatology:Question 23
The correct answer is A
Educational Objectives
Recognize the utility of aspirating a noninflamed joint in a patient with gout who is
asymptomatic between attacks.
Critique
The history suggests recurrent acute gout or possibly pseudogout. Aspiration of
synovial fluid from an asymptomatic joint is very likely to yield evidence for the
diagnosis. Even though this patient has not had an acute attack for 4 months, the
likelihood of finding crystals in the joint fluid is greater than 80%.
Serum uric acid levels may be elevated in patients who do not have gout, and
normal in those who do. Serum rheumatoid factor levels increase with age, and are
not specific. Imaging techniques, even an expensive MRI, would probably
demonstrate only osteoarthritis, a common finding in a patient of this age.

Rheumatology:Question 24
The correct answer is D
Educational Objectives
Recognize predictors of pulmonary hypertension in limited forms of scleroderma
(systemic sclerosis).
Critique
Pulmonary hypertension (PHT) is the leading cause of scleroderma-related deaths in
patients with the limited cutaneous form of scleroderma. The DLCO may be
significantly decreased for many years before the diagnosis of PHT is made, even in
the absence of interstitial fibrosis. In a controlled study, the mean pulmonary artery
pressure estimated on echocardiogram was only slightly higher in those who
developed PHT than in controls. Those with PHT had a mean DLCO of 52% of
predicted, 4.5 years before the diagnosis of PHT, compared with controls matched
for age, sex, and race who had mean DLCO values of 81% (P <0.0001). The
importance of suspecting, then diagnosing PHT in this form of the disease is that
potent pulmonary vasodilators and lung transplantation have improved patient well-
being and, perhaps, survival. In addition, an orally administered dual endothelin-
receptor antagonist (bosentan) has been shown to improve exercise capacity and
cardiopulmonary hemodynamics in patients with pulmonary arterial hypertension.
Bosentan (125 mg twice daily) is well tolerated. Nevertheless, the risk of death in
patients with PHT and systemic sclerosis is higher than in patients with primary PHT.
The pathology in this form of PHT is noninflammatory intimal proliferation of
medium-size pulmonary arteries, similar to that found in primary PHT. Patients with
this clinical picture rarely develop interstitial fibrosis on chest radiographs or clinical
examination and are not at risk (in contrast to those with diffuse disease) for
scleroderma renal crisis with malignant hypertension. Those with the limited
cutaneous form of scleroderma do not develop skin changes other than the
sclerodactyly. Telangiectasias are part of the limited scleroderma or CREST
syndrome (calcinosis, Raynauds phenomenon, esophageal dysmotility, sclerodactyly,
and telangiectasias) and are not the result of liver disease, portal hypertension, and
esophageal varices. Pulmonary function tests obtained when patients are first
diagnosed with any form of scleroderma are the best predictors of future risk of
severe lung disease.

Rheumatology:Question 25
The correct answer is D
Educational Objectives
Distinguish between hyperimmunoglobulinemia D with periodic fever syndrome and
other hereditary periodic fever syndromes.
Critique
The most likely diagnosis, on the basis of the clinical presentation, is the
hyperimmunoglobulinemia D with periodic fever syndrome (HIDS). Among the points
that are consistent with this diagnosis are the lifelong history of febrile episodes, the
duration of attacks, a family history consistent with autosomal recessive inheritance,
and the ethnic background.
Patients with HIDS often have a prodromal period with headache. Their abdominal
pain tends to be milder than that seen in familial Mediterranean fever or tumor
necrosis factor receptor-associated periodic syndrome (TRAPS), and is more likely to
be associated with diarrhea than with constipation. During attacks, patients with
HIDS may have oral ulcers and diffuse maculopapular rash involving the palms and
soles. In contrast, the typical rash of familial Mediterranean fever tends to be
localized to the foot or ankle, and the characteristic rash of TRAPS is migratory
erythema. The cryopyrin-associated periodic fevers usually present with an urticaria-
like rash. Cervical lymphadenopathy is much more common in HIDS than in familial
Mediterranean fever or TRAPS.
Although the patient’s pediatrician suggested the diagnosis of familial Mediterranean
fever, the equivocal response to colchicine does not support this diagnosis, and there
is no mention of genetic testing. Increasing the dose of colchicine in this patient
would be unlikely to alleviate the symptoms of the current attack or to prevent
future attacks. Administration of 3 mg of colchicine intravenously over 3 days might
actually lead to colchicine toxicity in a patient who is already receiving 1 .2 mg daily
by mouth.
Amyloidosis has not been reported in HIDS, and trace proteinuria is sometimes seen
during attacks of any of the hereditary periodic fever syndromes. Uveitis is not a
characteristic feature of HIDS.

Rheumatology:Question 26
The correct answer is D
Educational Objectives
Recognize complications of ankylosing spondylitis.
Critique
This patients history and symptoms are highly suggestive of a diagnosis of
ankylosing spondylitis or other spondyloarthropathy with significant axial skeletal
involvement. Following minimal trauma, the patient developed increased neck pain,
which provoked him for the first time to seek medical attention. Although he denied
neurologic symptoms, careful examination showed hyperreflexia and the presence of
Babinski’s sign, consistent with cord compression. Radiography or CT scan (or both)
of the cervical spine is the necessary next step.
In this setting, the clinician should be particularly alert to the possibility of an occult
fracture through a fused segment of the spine. If unrecognized, this complication can
lead to major neurologic deficits.
Although the other interventions may be useful for patients with other causes of neck
pain (analgesics and muscle relaxants and physical therapy for myofascial neck pain
or pain from osteoarthritis, and indomethacin for inflammatory neck and lower back
pain from spondyloarthropathy), these strategies would be initially inappropriate for
this patient, especially given his history of trauma and neurologic findings. No
intervention would also be an inappropriate strategy because of the risk of cord
compression from an unstable fracture, already suggested by this patients neurologic
findings.

Rheumatology:Question 27
The correct answer is A
Educational Objectives
Recognize posterior uveitis as one of the major complications of Behcets disease.
Critique
Among the symptoms of Behcets disease are oral ulcers, severe uveitis, and retinal
vasculitis. It can cause both anterior and posterior uveitis. The anterior uveitis is
usually highly symptomatic. By contrast, posterior uveitis or retinal vasculitis can
remain clinically silent until substantial retinal damage has occurred. The ocular
complications of Behçet’s disease, if untreated, can lead to blindness.
Giant cell arteritis can cause blindness, but the usual mechanism of visual loss is
anterior ischemic optic neuropathy—occlusion of the ophthalmic or posterior ciliary
arteries. Central retinal artery occlusion can also occur in giant cell arteritis. Uveitis,
however, is not associated with giant cell arteritis; furthermore, it is an exceptionally
rare complication of the other disorders listed. Microscopic polyangiitis and
polyarteritis nodosa occasionally cause eye disease—typically episcleritis, scleritis, or
anterior uveitis—but are not associated with retinal vasculitis. The typical ocular
manifestation of multiple sclerosis is optic neuritis.

Rheumatology:Question 28
The correct answer is C
Educational Objectives
Recognize the importance of arthrocentesis in the diagnosis and management of
knee effusions.
Critique
The best treatment would be to aspirate the knee and refer the patient for physical
therapy. It is important to remove collections of joint fluid, especially from knees,
because with motion the fluid pressure expands the joint capsule and can lead to
joint instability. Infection must be considered in a patient with monoarthritis, but in
the absence of systemic features, microbiologic examination of the fluid would be a
better choice than blood culture. The atrophy of the quadriceps muscle suggests a
chronic process, and attention must be paid to rebuilding muscle strength and
function.
Radiography of the knee would be useful to ascertain the extent of disease, and a
nonsteroidal anti-inflammatory drug might provide some relief of pain. It is likely,
however, that the problem is mechanical rather than inflammatory, and
acetaminophen would be a better choice.

Rheumatology:Question 29
The correct answer is B
Educational Objectives
Recognize the significance of bloody synovial fluid.
Critique
This patient has a fracture of the patella. Synovial fluid with a significant amount of
blood in it is not due to a traumatic arthrocentesis, which usually produces only a
tinge of blood. Bloody synovial fluid is usually indicative of a coagulopathy, tumor,
trauma, or Charcots joint. The description of the fall and the examination indicate
trauma to the patella rather than an anterior cruciate ligament tear. Linear patellar
fractures may not be detected on routine radiographs.
If this patient had osteogenesis imperfecta, it would have been symptomatic much
earlier in her life. Pigmented villonodular synovitis can produce bloody synovial fluid
but does not have an acute presentation.

Rheumatology:Question 30
The correct answer is A
Educational Objectives
Emphasize the importance of physical examination in diagnosis of hip pain.
Critique
The lateral point tenderness and full painless range of motion of the hip through all
dimensions except adduction suggest trochanteric bursitis. Although the patient’s
history of SLE and cirrhosis of the liver are predisposing factors for avascular
necrosis of the femoral head, this diagnosis is less likely in what many call the
‘greater trochanteric pain syndrome.’ Osteoarthritis and septic arthritis are
manifested by moderate pain (osteoarthritis) or severe pain (sepsis) on passive
motion, which would also be limited. Adductor muscle strain is painful with
abduction, not adduction, of the hip. In the presence of fever and markedly limited
range of motion, the possibility of sepsis would be high in the differential diagnosis.
Patients with or without SLE who have anticardiolipin antibodies are at greater risk
than control populations for avascular necrosis of the femoral head, and many times
the necrosis is asymptomatic.
Direct injection of lidocaine and corticosteroids into the area of the tenderness would
be indicated for this patient. In patients who are not helped significantly by such
injections, MRI is indicated to search for a tear or tendinitis of the gluteus medius or
avascular necrosis of the femoral head.

Rheumatology:Question 31
The correct answer is A
Educational Objectives
Diagnose Felty’s syndrome in a patient with rheumatoid arthritis and manage the
associated neutropenia.
Critique
This patient has Feltys syndrome. It was discovered during the 1970s that if the
active synovitis in patients with Feltys syndrome were treated, the neutropenia often
improved. When granulocyte colony-stimulating factor and granulocyte-macrophage
colony-stimulating factor became available, many patients could be treated
effectively, but in some the underlying rheumatoid arthritis flared while the profile of
their blood counts improved.
In this patient, little would be gained by treating the underlying disease because the
underlying synovitis, as assayed by clinical and laboratory measures, is inactive.
Therefore, splenectomy is the treatment of choice. Every patient without a spleen
must be vaccinated against the pneumococcus, because this encapsulated organism
is cleared by the spleen.
Reinitiating methotrexate therapy would not be useful because the disease in this
patient is inactive. Overwhelming infections could develop as a consequence of
etanercept therapy, and for this reason, use of the drug is contraindicated in
neutropenic patients. If one of the colony stimulating factors led to a flare of disease
in the past, even a small dose would be highly likely to cause another flare. There is
no evidence that plasmapheresis would help in neutropenia associated with Feltys
syndrome.

Rheumatology:Question 32
The correct answer is C
Educational Objectives
Manage familial cold urticaria.
Critique
This patients history is classic for familial cold autoinflammatory syndrome (FCAS;
formerly known as familial cold urticaria). There are currently no definitive
treatments for FCAS; nonsteroidal anti-inflammatory drugs are often used to control
the fever and joint stiffness.
Features consistent with the diagnosis include the requirement for generalized cold
exposure, the delay in onset of symptoms, the presence of an urticaria-like skin
rash, and the concomitant complaints of headache, chills, fever, extreme thirst, and
joint stiffness. Cold weather and air conditioning frequently trigger attacks in
patients with FCAS.
Allowing for the fact that symptoms may be minimal in warm climates, there appears
to be an autosomal dominant pattern of inheritance in this family. Laboratory studies
during an episode are consistent with systemic inflammation and a vigorous acute-
phase response. FCAS is caused by mutations in CIAS1, a gene that encodes the
protein cryopyrin. The interleukin-1 receptor antagonist anakinra has been shown to
be of benefit in the treatment of Muckle-Wells syndrome, which is also caused by
CIAS1 mutations; for this reason, the role of anakinra is currently being investigated
in the treatment of FCAS.
In FCAS, serum histamine levels are not elevated, and antihistamines are not
generally effective in controlling the rash. The clinical history is not consistent with
cryoglobulinemia.
Patients with the acquired form of cold urticaria may develop hypotension and
angioedema, but those with FCAS do not; hence there is no role for epinephrine in
the management of these patients. Patients with acquired cold urticaria develop
localized urticaria on localized cutaneous exposure to cold and respond positively to
the ice-cube challenge test; patients with FCAS do not.

Rheumatology:Question 33
The correct answer is D
Educational Objectives
Recognize the clinical features of reactive arthritis.
Critique
This patient developed generalized swelling and pain in one toe in the absence of any
definable underlying disease such as long-standing diabetes. As an isolated finding,
particularly with the radiographic and MRI findings described, infection certainly is a
reasonable diagnosis. His failure to respond to antibiotics, however, and most
especially the negative cultures at the time of amputation strongly indicate that
infection was not a source of his toe pain. His symptoms are suggestive of uveitis
and urethritis and support the unifying diagnosis of reactive arthritis, in which the
joint disease is dactylitis.
Dactylitis can cause chronic diffuse inflammation of entire digits, particularly the
toes, and can lead to erosive bony changes. Option D is correct because this answer
describes other clinical features associated with complicated reactive arthritis. The
rash on the palms and soles in reactive arthritis is known as keratoderma
blennorrhagicum and the penile lesions as circinate balanitis.
Although many of the clinical features of reactive arthritis, including sacroiliitis,
spondylitis, and uveitis, are related to HLA-B27, testing for this allele is generally not
useful diagnostically. Furthermore, it can never establish with certainty the diagnosis
of spondyloarthropathy in general or reactive arthritis in particular because of the
high frequency of HLA-B27 positivity in the general population without disease.
The diagnosis of osteomyelitis is incorrect because of this patient’s clinical course
(failure to respond to antibiotics, lack of organisms at the time of amputation) and
the observation that osteomyelitis does not generally “spread” to contiguous tissues.
Option C is incorrect because a patient with reactive arthritis would be expected to
have inflammatory back pain, which is typically worse in the morning and improves
as the day progresses. The back pain described in option C is more likely to be
mechanical because it worsens with activity and as the day progresses.
Option E is incorrect because the anterior chamber is the most commonly involved
part of the uvea in reactive arthritis; patients with posterior uveitis are more likely to
have other disorders, such as sarcoidosis or toxoplasmosis.

Rheumatology:Question 34
The correct answer is D
Educational Objectives
Recognize cutaneous ulcerations associated with medium-vessel vasculitis.
Critique
The patient has several features strongly suggestive of a medium-vessel vasculitis,
the most likely cause of which in her case is polyarteritis nodosa. Features
suggestive of this disease are the lower extremity ulcers and the weakness in her
extremities, which are consistent with mononeuritis multiplex-nerve infarctions that
result from vasculitic involvement of the vasa nervorum.
There are no symptoms suggestive of lymphoma with a paraneoplastic syndrome (B
symptoms:
adenopathy, unexplained weight loss of >10% in the last 6 months, unexplained
fever >38 °C [100.4 °F] in the previous month, recurrent drenching night sweats in
the previous month) suggesting that specific diagnosis. Takayasus arteritis is a large-
vessel vasculitis more likely to involve the aorta and its branches than the medium-
sized arteries of the lower extremities; moreover, Takayasus arteritis does not cause
vasculitic neuropathy. Systemic lupus erythematosus can be excluded with
confidence in this case because testing for the antinuclear antibody has been
repeatedly negative. Finally, Kawasakis disease, or mucocutaneous lymph node
syndrome, is a pediatric disease that occurs nearly exclusively in patients younger
than 6 years old, is associated with adenopathy, high fevers, erythema of the
mucous membranes, and—in severe cases—aneurysms of the coronary arteries.

Rheumatology:Question 35
The correct answer is C
Educational Objectives
Diagnose relapsing polychondritis.
Critique
This patient has relapsing polychondritis, which can be diagnosed by the presence of
three of six clinical signs. The six criteria include bilateral auricular chondritis;
nonerosive, seronegative polyarthritis; nasal chondritis; ocular inflammation;
respiratory tract chondritis; and cochlear or vestibular dysfunction. The best way to
document the diagnosis and avoid future uncertainty is to obtain tissue for histology.
Polychondritis does occur with several rheumatic diseases, including systemic lupus
erythematosus.
Given the symptom complex of fatigue, malar redness, and synovitis in a young
woman, screening for systemic lupus erythematosus by an antinuclear antibody test
would be prudent. A positive result does not indicate that a patient has systemic
lupus erythematosus, but a negative result makes the diagnosis extremely unlikely.
Rheumatoid arthritis is possible, but testing for rheumatoid factor would not shed
light on the diagnosis, which is made on clinical grounds (one criterion for
classification of rheumatoid arthritis being persistence of synovitis for at least 6
weeks). Although the patient recently returned from the Caribbean, she does not
have gastrointestinal symptoms; therefore a stool culture is not necessary. Given the
short duration (2 weeks) of joint aching, it is unlikely that radiography would reveal
any abnormality.

Rheumatology:Question 36
The correct answer is B
Educational Objectives
Diagnose septic arthritis in a patient with pre-existing rheumatoid arthritis.
Critique
Aspiration to rule out infection is imperative and must be done before any other
interventions are undertaken. If infection is present, aspiration of the joint will yield
cloudy synovial fluid. Patients with rheumatoid arthritis or other pre-existing joint
damage have an increased susceptibility to infection of the joint; the rapid onset of
severe joint pain and the physical finding of immobility should alert the examiner to
this possibility. In patients on immunosuppressive therapy, the absence of fever or
other systemic manifestations of sepsis is not unusual. Injection of corticosteroids
should be withheld until a leukocyte count, Gram stain, and culture have been
performed, looking for a staphylococcal or other bacterial infection.

Rheumatology:Question 37
The correct answer is C
Educational Objectives
Recognize the link between hypertrophic osteoarthropathy and lung cancer.
Critique
This patients history of pain in his ankles and lower extremities together with
periosteal new bone formation on radiographs is characteristic of hypertrophic
osteoarthropathy. Almost pathognomonic is the observation that elevation of the
extremities above the head brings instant relief. A close examination of the hands for
true clubbing” is also indicated.
Whenever hypertrophic osteoarthropathy is present, it is critical to search for an
associated disease. Neoplasm (pulmonary, gastrointestinal, lymphoma) and infection
are two important conditions in which hypertrophic osteoarthropathy can be seen. In
adult patients with hypertrophic osteoarthropathy, lung cancer must be ruled out. It
is essential to obtain a smoking history. A chest radiograph should be the first step.
If results are equivocal, a CT scan should be obtained.
This condition can also exist in chronic hypoxic states, in progressive, severe
peripheral arterial insufficiency, and in inflammatory bowel disease. Children with
cystic fibrosis and arterial oxygen desaturation can develop hypertrophic
osteoarthropathy in wrists, knees, and ankles. Occasionally a mild synovitis is
present.
Although the cause of hypertrophic osteoarthropathy is unknown, vascular
endothelial growth factor (one study) and growth hormone-releasing hormone (one
study) have been found to be elevated in patients with hypertrophic arthropathy. It
is not known whether treatment of the underlying condition (resection of lung
cancer, for example) will result in normalization of these factors.
NSAIDs can give relief in this syndrome but do not result in a diagnosis, which is the
most important immediate objective. Thyroid function tests could reveal a mild
hypothyroid state because the patient has not taken his levothyroxine for 2 weeks,
but the arthropathy of hypothyroidism is not associated with periosteal new bone
formation. A tuberculin skin test is appropriate if a chest radiograph shows infiltrates
that clearly are not malignant but is not an appropriate initial course in this case. A
radionuclide bone scan would show excessive uptake in the areas of periosteal new
bone formation, but would not help diagnostically.

Rheumatology:Question 38
The correct answer is B
Educational Objectives
Diagnose methotrexate-induced lung disease.
Critique
There are a number of toxicities associated with the use of disease-modifying
antirheumatic drugs in rheumatoid arthritis. For methotrexate, they include infection
and hypersensitivity pneumonitis. Methotrexate lung disease presents with dry
cough, shortness of breath, and fever. It is most often seen in the first 6 months of
therapy and is evidenced by a diffuse interstitial pattern on radiographs. The acute
mortality rate is 17%. Treatment is to discontinue the methotrexate and institute
corticosteroid therapy.
Rheumatoid lung disease can present in four patterns: pleural effusions, pulmonary
fibrosis, rheumatoid lung nodules, and Caplan’s syndrome (pneumoconiosis
associated with lung rheumatoid nodules). Bacterial pneumonias generally are
associated with sputum production, and reactivation of pulmonary tuberculosis would
not show this radiographic pattern. Although interstitial lung disease associated with
rheumatoid arthritis might be part of the radiographic differential diagnosis, it would
not appear suddenly, as did the symptoms in this patient.

Rheumatology:Question 39
The correct answer is D
Educational Objectives
Recognize the conditions that can cause arthritis with pyoderma gangrenosum and
the genetic tests to distinguish among them.
Critique
Although genetic testing can be a useful adjunct in the diagnosis of patients who
have arthritis with pyoderma gangrenosum, the clinician should rely on the patients
history of associated symptoms. The differential diagnosis of arthritis and pyoderma
gangrenosum includes inflammatory bowel disease, Behçet’s disease, rheumatoid
arthritis, and the seronegative axial or peripheral arthritides. The history of
asymmetric destructive arthritis, cystic acne, pyoderma gangrenosum, and pathergy
are consistent with pyogenic arthritis with pyoderma gangrenosum and acne (PAPA
syndrome), which is caused by dominantly inherited mutations in PSTPIP1.
Some patients with inflammatory bowel disease have mutations inNOD2, but the
clinical points arguing against this diagnosis are the lack of gastrointestinal
symptoms, the onset of arthritis at age 4 years, and the apparent three-generation
inheritance in this family.
Behçet’s disease can present with arthritis, acneiform lesions, pyoderma
gangrenosum, and pathergy, and is associated with the HLA-B51 antigen. However,
the arthritis of Behçets disease tends not to be erosive, and the patient has no
history of ocular or oral inflammation.
Rheumatoid arthritis can present with arthritis and pyoderma gangrenosum and is
associated with an increased frequency of HLA-DR1 and -DR4, but the pattern of
joint involvement in this patient would be very atypical for rheumatoid arthritis.
Myeloid leukemia can present with pyoderma gangrenosum, but other aspects of the
case, including the arthritis, cystic acne, and three-generation inheritance, would be
very unusual.

Rheumatology:Question 40
The correct answer is D
Educational Objectives
Treat psoriatic spondyloarthropathy.
Critique
This patient clearly has advanced psoriatic skin and joint disease. Methotrexate is the
most commonly prescribed agent to treat both cutaneous and articular disease. It is
almost always used in addition to nonsteroidal anti-inflammatory drugs (NSAIDs) in
the management of significant psoriatic joint disease and as a single agent in the
treatment of isolated cutaneous disease.
In many patients meaningful improvement in skin and joint symptoms will occur
during treatment with methotrexate. Patients with psoriasis who take methotrexate
should be cautious about alcohol consumption because alcohol enhances the risk of
methotrexate-induced hepatic toxicity, which is higher in patients with psoriatic
arthritis than in those with rheumatoid arthritis who use this drug.
NSAIDs are important in the initial treatment of inflammatory arthritis and might
confer rapid relief of some, but probably not most, of this patient’s joint symptoms.
This class of agents alone would not, however, treat her skin disease. Similarly,
although topical agents might be effective in managing the skin disease, they would
not effectively treat articular disease.
Although hydroxychloroquine is an effective agent in the treatment of cutaneous
lupus, it is contraindicated in psoriasis, because it can aggravate the underlying skin
disease. Etanercept has recently been approved by the U.S. Food and Drug
Administration for the treatment of psoriatic arthritis and is often noted to improve
cutaneous disease as well. This drug appears to be effective and safe, but it is
expensive and at present should probably be reserved for patients whose disease is
refractory to less expensive, more time-honored agents. The other available biologic
disease-modifying antirheumatic drugs (infliximab and adalimumab) are currently
under investigation for use in psoriasis and psoriatic arthritis.

Rheumatology:Question 41
The correct answer is A
Educational Objectives
Diagnose and treat Buergers disease (thromboangiitis obliterans).
Critique
The patient should quit smoking completely. He has Buerger’s disease
(thromboangiitis obliterans), which occurs primarily in young men and is strongly
associated with tobacco use, particularly cigarette smoking. It tends to involve the
medium-sized arteries and veins of the distal extremities, classically at the level of
the wrists and ankles. Buergers disease can lead to severe digital ischemia with
gangrene but very seldom involves any internal organs. The proximal vessels are
normal in Buergers disease.
The only known effective therapy for Buergers disease is smoking cessation. Patients
who continue to smoke are at high risk of having to undergo amputation.
Immunosuppressive, anticoagulant, and thrombolytic agents are not effective in
Buergers disease.

Rheumatology:Question 42
The correct answer is C
Educational Objectives
Recognize that criteria for joint replacement surgery are clinical, not radiographic.
Critique
The best advice for this patient is to put modest limits on trauma to his knees as he
has done, and to have formal instruction in exercises to strengthen muscles and
protect joints. He should be evaluated at intervals (two to three times per year) to
monitor pain and knee function.
The indications for total knee replacement in patients with osteoarthritis are pain and
limitation of function to a degree sufficient to interfere with enjoyment of life.
Patellofemoral osteoarthritis is not an indication for joint replacement. Many older
people have meniscal tears that are asymptomatic. This patient has had progression
of symptoms (reflected in his reduced activity) and radiographic evidence of cartilage
degradation. He is quite active, however, and surgery would not likely increase that
amount of activity. A recent double-blind study suggests that shaving the cartilage
and washing out the joint in elderly patients is not as useful as had been thought.

Rheumatology:Question 43
The correct answer is D
Educational Objectives
Recognize that lupus anticoagulant tests in patients taking heparin or warfarin are
not valid.
Critique
It is not possible to determine from the tests done whether she has a lupus
anticoagulant. Lupus anticoagulant assays are clotting times, which are affected by
both heparin and warfarin. Before the assays are done for this patient, the plasma
should be treated with heparinase to remove heparin.
New classification criteria for the antiphospholipid antibody syndrome have been
adopted and include thrombosis and adverse pregnancy outcomes in the setting of
either a lupus anticoagulant or anticardiolipin antibody. One of the criteria for
antiphospholipid antibody syndrome, however, is that the woman has had more than
one first-trimester loss. This patient does not meet criteria for antiphospholipid
antibody syndrome. Anticardiolipin antibody levels must be in the medium- to high-
positive range before the disease is considered clinically important.

Rheumatology:Question 44
The correct answer is B
Educational Objectives
Recognize infection of the sacroiliac joint and the possibility of soft-tissue extension.
Critique
Because it resolves soft tissues best, MRI of the spine is the modality of choice.
Infections of the fibrocartilaginous joints, including the sacroiliac joint, are more
frequent in injection drug users with or without HIV infection. The symptoms often
suggest a low back problem, but the tenderness in the buttock and pain on pelvic
compression in this patient strongly suggest a sacroiliac source. Such infections may
involve the surrounding tissues.
Once the extent of the infection has been demonstrated, CT-guided aspiration of the
joint would be appropriate.

Rheumatology:Question 45
The correct answer is B
Educational Objectives
Manage pain in severe osteoarthritis.
Critique
As many as 80% of patients older than 75 years show radiologic signs of
osteoarthritis. Pharmacologic management guidelines should be based on overall
efficacy, toxicity profiles of possible drugs, and cost. Although acetaminophen in full
doses is often prescribed early for patients with osteoarthritis to relieve pain, there is
increasing evidence that it is not as effective as standard nonsteroidal anti-
inflammatory drugs (NSAIDs) in symptomatic treatment of osteoarthritis. The best
therapy for this patient would be daily administration of long-acting morphine.
Because this patient has had two gastrointestinal hemorrhages, standard NSAIDs
cannot be used.
Rofecoxib would be inappropriate therapy because of his renal insufficiency, and the
cyclooxygenase-2 (COX-2) inhibitors have the same inhibitory effect on renal blood
flow as the standard COX-1/COX-2 drugs. Although randomized controlled trials
indicate that glucosamine supplementation can provide some degree of pain relief in
osteoarthritis, it is unlikely that it could assuage this patients severe pain.
A sleep clinic might be appropriate if standard therapies are ineffective; it is
appropriate, however, first to prescribe a more powerful analgesic to achieve the
goal of providing better sleep for this man.

Rheumatology:Question 46
The correct answer is E
Educational Objectives
Recognize palpable purpura as a symptom of small-vessel vasculitides.
Critique
The location of the patient’s rash, over the buttocks and lower extremities, is a
typical distribution for palpable purpura. This type of skin rash is caused by vasculitic
involvement of small blood vessels (usually the post-capillary venules). Microscopic
polyangiitis, which is commonly associated with anti-neutrophil cytoplasmic
antibodies, involves small (and occasionally medium-sized vessels) and may present
with palpable purpura. Microscopic polyangiitis is also usually associated with internal
organ involvement such as glomerulonephritis, alveolar hemorrhage, or vasculitic
neuropathy. In this case, the onset of the purpura 10 days after the patient started
taking antibiotics, combined with the absence of systemic symptoms, is more
suggestive of hypersensitivity vasculitis caused by a drug reaction.
Because temporal (giant cell) arteritis involves large and medium-sized arteries
exclusively, it would not be associated with this kind of presentation. Polyarteritis, a
disease of medium-sized blood vessels, does not have palpable purpura as a
presenting symptom. Acute leukemia, with its associated thrombocytopenia, is often
marked by flat purpura and bleeding from the gums, but rarely by palpable purpura.
Churg-Strauss vasculitis can have associated skin lesions, but usually occurs in the
context of a history of asthmatic bronchitis.

Rheumatology:Question 47
The correct answer is C
Educational Objectives
Diagnose and treat anserine bursitis, a common malady in patients with
osteoarthritis of the knee.
Critique
The patient has anserine bursitis; the maneuver with the knee semiflexed helps
confirm the diagnosis. Corticosteroid injection at the bursal site almost always
provides relief of pain. Often, knee pain attributed to even severe osteoarthritis of
the knee disappears after treatment of the anserine bursitis. In addition, patients
should adhere to a regimen of isometric quadriceps exercises, and if applicable, to
weight reduction.
In anserine bursitis, the diagnosis rests on the finding of focal tenderness on the
upper inner tibia about 5 cm distal to the medial articular line of the knee. Patients
are usually middle-aged or older and often have knee osteoarthritis, but the problem
can occur in active young people also. Usually there is no redness, swelling, or
increased warmth at the painful site. It may be that the underlying problem is strain
of the pes anserine tendon rather than true bursitis. Because the corticosteroid is
injected into soft tissue, risk of tendon rupture is minimal or nil. Skin atrophy can
occur.
Increased doses of a nonsteroidal anti-inflammatory drug or acetaminophen may
diminish the pain somewhat, but much discomfort will persist, especially with stair
climbing and at night. Systemic corticosteroids are not effective for anserine bursitis
and are not necessary, given the efficacy of local injections. The diagnosis is clear
enough, so that an MRI would not be of help. If the injection does not afford relief,
stretching exercises might be useful.

Rheumatology:Question 48
The correct answer is A
Educational Objectives
Recognize that systemic lupus erythematosus predisposes to premature
atherosclerosis and
myocardial ischemia.
Critique
The patients serum troponin or creatine kinase level should be determined. The
initial concern is whether the chest pain and electrocardiographic changes are caused
by pericarditis or a myocardial process. The lack of positional and pleuritic
components to the pain makes pericarditis less likely. Some studies of patients with
systemic lupus erythematosus have shown that cardiovascular disease secondary to
accelerated atherosclerosis is a major cause of death. If myocardial infarction is
suspected, it is more likely to be caused by atherosclerosis than by lupus vasculitis
or myocarditis. Although antiphospholipid antibodies were not found in the patient’s
laboratory studies, pulmonary embolus cannot be excluded. A ventilation-perfusion
scan or high-resolution CT scan of the chest may be indicated, but cardiac enzyme
levels should be obtained first. An echocardiogram would not be helpful because no
pericardial rub was heard, and the electrocardiogram did not suggest pericarditis.
The lung examination revealed no abnormalities and the arterial blood gases are
normal. Because there is little concern that pulmonary fibrosis is present, pulmonary
function tests are not indicated.

Rheumatology:Question 49
The correct answer is D
Educational Objectives
Recognize signs and symptoms of mechanical injury.
Critique
This patient probably has a mechanical injury to his knee, such as a torn meniscus,
which could cause the locking; the physical examination is consistent with this
diagnosis. Arthroscopy will be both diagnostic and therapeutic. A nonsteroidal anti-
inflammatory agent may be helpful in the short term, but there are other agents
besides celecoxib that are less expensive and equally effective.
Although his diuretic may be the cause of his elevated uric acid, his symptoms are
not consistent with gout, and there is no reason to discontinue the
hydrochlorothiazide or begin allopurinol. Aspiration and injection can provide
temporary relief but will not correct the underlying mechanical problem. Small
nodules in the olecranon bursae are a common sequela of trauma.

Rheumatology:Question 50
The correct answer is A
Educational Objectives
Recognize the radiographic findings of homozygous sickle cell anemia.
Critique
The hallmarks of sickle cell disease include bone infarcts, periosteal elevation, and
avascular necrosis of bone. These findings are often associated with a history of a leg
ulcer and osteonecrosis of the humeral head. Femoral head osteonecrosis is also
very common in adults with sickle cell disease.
After plain radiographs, MRI is often needed to evaluate the extent of bone
involvement. Between sickle cell crises, most patients are asymptomatic.
Symptomatic osteonecrosis of the hip in sickle cell disease has a high likelihood of
progressing to femoral head collapse, necessitating surgical intervention.
Symptomatic aseptic loosening after total hip replacement arthroplasty is much more
common in these patients than in the general population. Interestingly, in contrast to
corticosteroid-induced osteonecrosis, apoptotic osteocytes are not seen in bone
samples. Hemophilic arthropathy generates bone and cartilage erosions very similar
to those of rheumatoid arthritis. Chondrocalcinosis is a complication of
hemochromatosis. Although severe osteoarthritis with chondrocalcinosis in multiple
joints is found in patients with long-standing disease, it would be unusual in a young
adult. Subperiosteal bone resorption is a characteristic skeletal finding in response to
long-term excessive secretion of parathyroid hormone. Periosteal new bone
formation is rarely seen in the femoral head/neck area, but is a characteristic of
chronic ischemia from cardiopulmonary or local vascular causes.

Rheumatology:Question 51
The correct answer is B
Educational Objectives
Identify hepatitis C infection as the cause of mixed cryoglobulinemia.
Critique
This patient has mixed cryoglobulinemia, probably associated with hepatitis C, which
her pregnancy-related blood transfusions put her at risk for contracting. Hepatitis C
causes as many as 90% of cases of cryoglobulinemia, previous known as “essential”
mixed cryoglobulinemia (the “mixed” part of the name for this disorder derives from
the fact that the condition is associated with both 1gM and lgG antibodies). The lgG
antibodies are generally polyclonal, but the 1gM antibodies are often monoclonal and
have a specificity for the Fc portion of the lgG molecule. This specificity is the
definition of a rheumatoid factor. Consequently, nearly all patients with
cryoglobulinemia associated with hepatitis C infections test positive for rheumatoid
factor. Many are positive for antinuclear antibodies, as well.
These autoantibodies frequently lead to misdiagnoses of rheumatoid arthritis or
systemic lupus erythematosus. Mixed cryoglobulinemic vasculitis is an immune
complex-mediated vasculitis leading to the consumption of complement proteins
(and their deposition in lesions). Serum C4 levels are usually decreased
disproportionately to C3 levels in this disease.

Rheumatology:Question 52
The correct answer is C
Educational Objectives
Diagnose and treat pes planus.
Critique
It is important to have the patient stand and to view his feet from behind. Patients
with flexible flat feet develop a longitudinal arch while sitting with the feet dangling
or by rising on their tiptoes; viewed from the rear, the patient can be seen to have a
collapsed arch, heel valgus, pronated feet, and often a displaced Achilles tendon
(does not bisect the foot equally). Referral to a podiatrist for custom fabrication of
orthotics is warranted. Store-bought orthotics are not nearly as useful.
Many people are cured of ankle or knee pain after correction of foot mechanics. If
there is Achilles tendon contracture, stretching exercises are recommended.
Although severe pronation can result in posterior tibialis tendinitis, anti-inflammatory
agents do not offer relief.
Better-quality shoes would afford better support, but would not in the long run give
the most relief. Support stockings would not address the problem at all. Spending
less time on his feet would help but is not always a viable alternative.

Rheumatology:Question 53
The correct answer is C
Educational Objectives
Recognize that antiphospholipid antibodies cause both venous and arterial
hypercoagulability.
Critique
Antiphospholipid antibody syndrome causes both venous and arterial
hypercoagulability. Most arterial events are transient ischemic attacks and strokes,
which are frequently embolic, rather than caused by thrombosis in situ. Valvular
vegetations are the usual cause, with the mitral valve being most frequently
affected. This condition may be diagnosed with transesophageal echocardiography,
which is more sensitive than transthoracic echocardiography.
In patients with antiphospholipid antibody syndrome, several retrospective studies
have suggested that an INR of 3.0 is preferable to lower-intensity anticoagulation.
Thrombocytopenia can occur in patients with antiphospholipid antibody syndrome;
therefore thrombotic thrombocytopenic purpura need not be invoked. Although
patients with systemic lupus erythematosus may have antiphospholipid antibodies,
this patient has no other symptoms to suggest this disorder.
Hemorrhage is unlikely with an INR of 2.1 and would have been detected on the
initial CT. Paradoxical embolism through a patent foramen ovale is less likely,
because the patient has no current symptoms or signs of deep venous thrombosis.

Rheumatology:Question 54
The correct answer is E
Educational Objectives
Recognize that inclusion body myositis is unresponsive to treatment.
Critique
The insidious onset of symptoms, the patients relatively low serum creatine kinase
level, and mixed myopathic and neurogenic findings on muscle biopsy are
characteristic of inclusion body myositis, the most common form of myositis in
patients older than 60 years. The muscle biopsy is diagnostic. This disease responds
poorly, if at all, to therapies usually effective in inflammatory myositis. It is wiser to
discontinue treatment than to expose the patient to additional potentially harmful
and expensive treatments.

Rheumatology:Question 55
The correct answer is E
Educational Objectives
Distinguish among polymyalgia rheumatica and fibromyalgia and other serious
organic diseases.
Critique
This complicated case illustrates several important points. Most important is that an
erythrocyte sedimentation rate as high as 98 mm/h must be considered a sign of
serious organic disease, and its cause must be sought, particularly when a significant
anemia is present.
Polymyalgia rheumatica and fibromyalgia are the most common diffuse pain
syndromes in the elderly. Fibromyalgia can exist in a patient who has other more
serious organic pathology, and it is unusual for polymyalgia rheumatica not to
improve with such a high daily dose of prednisone. Possible diagnoses include renal
cell carcinoma, lymphoma, granulomatous diseases (including tuberculosis), and
asymptomatic vasculitis.
Nortriptyline and aerobic exercise programs are useful for fibromyalgia in some
patients, as is the use of selective serotonin reuptake inhibitors. Recent data indicate
that methotrexate fails to effectively substitute for corticosteroid therapy in giant cell
arteritis.

Rheumatology:Question 56
The correct answer is A
Educational Objectives
Recognize the association between antineutrophil cytoplasmic autoantibodies and
pulmonary-renal syndromes.
Critique
This patient has a pulmonary-renal syndrome, that is, the occurrence of alveolar
hemorrhage in association with glomerulonephritis. The differential diagnosis of this
presentation is fairly short and includes as its principal components microscopic
polyangiitis, Wegener’s granulomatosis, Churg-Strauss syndrome, systemic lupus
erythematosus, and Goodpastures syndrome (anti-glomerular basement membrane
antibody disease). Conditions associated with antineutrophil cytoplasmic
autoantibodies (ANCA), particularly microscopic polyangiitis and Wegeners
granulomatosis, are the types of diseases most likely to cause this presentation
(several times more likely, for example, than Goodpastures syndrome). In such
cases of ANCA-associated pulmonary-renal syndromes, the patient almost always
has antibodies to either myeloperoxidase (MPO-ANCA, which cause a perinuclear [P-
ANCA] pattern of immunofluorescence on testing of the patients serum) or
proteinase 3 (PR3-ANCA, which cause cytoplasmic [C-ANCA] immu nofluorescence).
Anti-Ro (SS-A) and anti-La (SS-B) antibodies are associated most strongly with Sj
grens syndrome. Anti-Sm antibodies are highly specific (but have poor sensitivity)
for systemic lupus erythematosus, occurring in approximately 25% of patients with
that disease. Antiribonucleoprotein antibodies often occur in a subset of patients with
systemic lupus erythematosus that is sometimes said to be a separate disorder:
“mixed connective tissue disease.’

Rheumatology:Question 57
The correct answer is C
Educational Objectives
Recognize that presence of monosodium urate crystals in synovial fluid does not rule
out septic arthritis.
Critique
The pattern of migrating polyarthritis, the genitourinary symptom, and what sounds
like tenosynovitis suggest strongly that the patient has disseminated gonococcal
infection; this diagnosis takes precedence over any other cause of arthritis.
Appropriate treatment is ceftriaxone. Chances of obtaining a positive gonococcal
culture from blood and synovial fluid are modest (approximately 50%).
The presence of monosodium urate crystals in the synovial fluid is likely due to the
action of enzymes generated in the inflammatory (septic) response. Most tophi are in
synovium, and this man very likely has gout in addition to gonococcal disease. The
enzymes released from inflammatory cells loosen crystals from the tophi (“enzymatic
strip mining”). The usual presentation of acute gout is monoarthritis.
Low-dose colchicine is used to prevent attacks of gouty arthritis but will do little for
acute gouty arthritis and less for infectious arthritis. Allopurinol would not be given
unless the serum uric acid concentration were known, nor would it usually be given
after one episode of gouty arthritis.

Rheumatology:Question 58
The correct answer is D
Educational Objectives
Recognize the need for effective long-term anticoagulation therapy for serious
arterial or venous thromboses due to antiphospholipid antibody syndrome.
Critique
Once a patient has had one event related to antiphospholipid antibody, like this
patients pulmonary embolism, another thrombosis is likely unless effective long-term
(maybe life-long) anticoagulation continues. Although there is debate about the
exact degree of anticoagulation therapy necessary (some authorities recommend a
target INR of 3.0; others recommend 3.5), an INR of 1 .9 is clearly too low.
Withdrawing warfarin appears to increase recurrence risk even more for several
months. Aspirin is unlikely to prevent a subsequent event, and the present warfarin
dosage is not adequate to achieve a therapeutic INR.

Rheumatology:Question 59
The correct answer is B
Educational Objectives
Diagnose interstitial lung disease.
Critique
This patient has interstitial lung disease, which occurs in approximately 10% of
patients with polymyositis, more so in those with Raynaud’s disease.
The clinical features and radiographic appearance are inconsistent with aspiration,
Pneumocystis infection, or bronchiolitis with organizing pneumonia. Methotrexate-
induced lung disease usually has an abrupt onset, often with fever.

Rheumatology:Question 60
The correct answer is C
Educational Objectives
Recognize factors other than inflammatory disease that can affect the erythrocyte
sedimentation rate.
Critique
The erythrocyte sedimentation rate (ESR), although it has been relied upon by
generations of physicians for estimating the extent of inflammation in the body, is
being supplanted by the C-reactive protein (CRP) test for this purpose. The ESR is
affected by many factors, principally the albumin:globulin ratio; the lower this ratio,
the higher the ESR. Therefore, in the presence of chronic liver disease characterized
by hypoalbuminemia and hyperglobulinemia, the ESR will be elevated with or without
systemic inflammation. Because CRP is synthesized by liver cells, in end-stage liver
disease the level could be falsely low, but if elevated, it is still a useful inflammatory
index. CRP is not directly affected by hyperglobulinemia.
Patients with active rheumatoid disease without hypersplenism or drug toxicity are
likely to have elevated leukocyte counts. This patient has the strong possibility of
having hypersplenism and leukopenia from her liver disease; if the spleen were
enlarged, there would be no clinically appropriate way of differentiating between liver
disease and Feltys syndrome as a cause of the mild leukopenia.
Serum haptoglobin determinations can be useful for assaying the extent of hemolysis
in anemia, but it is likely that this woman’s anemia is caused by chronic disease or
by blood loss, the latter from gastric or duodenal irritation by drugs or by oozing
from esophageal varices. In the absence of liver disease there is a strong inverse
correlation between the ESR and the hemoglobin concentration.
Although methotrexate toxicity in the liver can cause abnormalities of liver enzyme
levels, the elevation in this patient is small, and early cirrhosis is likely the cause.
Rheumatologists urge patients to not to drink alcohol or to drink very little while
taking methotrexate, a drug that directly affects liver metabolism, increasing
collagen production by hepatic cells. Folic acid, useful for helping to minimize the
development of apthous ulcers in patients taking methotrexate, has not been
reported to minimize liver toxicity.

Rheumatology:Question 61
The correct answer is C
Educational Objectives
Recognize the distinctive clinical presentation of Wegener’s vasculitis.
Critique
Antineutrophil cytoplasmic antibodies (ANCA) are found in the overwhelming
majority of patients with Wegeners granulomatosis. Antibodies to proteinase 3
impart a pattern of diffuse cytoplasmic fluorescence when studied by indirect
immunofluorescence and occur in several conditions (such as polyarteritis) in
addition to some cases of Wegeners granulomatosis.
Antiglomerular basement membrane antibodies are present in patients with
Goodpastures syndrome. These patients do not have recurrent sinusitis or
involvement of the nasal septum.
Biopsy diagnosis of granulomatous vasculitis is definitive, but biopsy of this patients
ulcerated nasal septum would not be helpful and might lead to septal perforation.
Chest radiographs are usually abnormal in this condition and would precede biopsy
but would not differentiate Wegeners granulomatosis from other lung diseases that
cause infiltrates, nodules, or cavitation. Even if a sputum culture were positive, it
would not explain the upper respiratory tract changes.
As is the case in many forms of vasculitis, most patients who have Wegeners
granulomatosis experience an insidious onset of disease, with malaise, weight loss,
and proximal respiratory symptoms. The mean age of onset is the sixth decade, and
the condition occurs with equal frequency in men and women.
Approximately 30% of patients have hearing loss, and septal ulceration can lead to
collapse of the nasal bridge and the characteristic “saddle nose’ deformity. Eye
involvement (episcleritis, scleritis) and proptosis due to retrobulbar granulomas
occur in 50% of patients. A nonresolving pneumonia, hemoptysis, or pleuritic pain
can be among the first indications of disease. Leukocytosis and a high erythrocyte
sedimentation rate are common, and patients often test positive for rheumatoid
factor.

Rheumatology:Question 62
The correct answer is C
Educational Objectives
Recognize that ‘chronic Lyme disease’ is a nonentity.
Critique
The patient probably had Lyme disease 10 months ago. Antibiotic treatment of
erythema migrans is usually curative. Her symptom complex is likely due to
myofascial pain syndrome. The best treatment for that entity is exercise (stretching,
light weights, swimming, or equivalent) and an agent to facilitate restorative sleep.
Many patients with chronic pain syndromes have an element of depression, with
associated fatigue and an ability to see things (swollen joints or limbs) an examiner
does not appreciate on physical examination. Extended post-Lyme disease antibiotic
therapy does not help.
Certainly joint symptoms (usually monoarthritis, usually of the knee) can occur
months after the rash of Lyme disease. Central nervous system manifestations of
Lyme disease are characterized mainly by encephalopathy and polyneuropathy and
are usually treated with ceftriaxone. In this patient, the muscle pain is due to muscle
tightness and spasm and muscle strength is full; therefore, muscle biopsy is not
warranted. Patients with myositis usually have muscle weakness and an elevation in
serum creatine kinase levels.
Prednisone and nonsteroidal anti-inflammatory drugs are not appropriate therapy for
chronic pain syndromes. Patients may perceive initial benefit but it rarely lasts and
the risks are substantial.

Rheumatology:Question 63
The correct answer is B
Educational Objectives
Recognize the association of antihistidyl-tRNA synthestase with interstitial lung
disease in polymyositis.
Critique
Myositis-specific autoantibodies occur in approximately 30% of patients with
inflammatory myositis. Antibodies specific for aminoacyl-tRNA synthetases are
associated with interstitial lung disease, arthritis, and a hyperkeratotic rash on the
hands.
Antibodies directed against signal recognition peptide are found in classic
dermatomyositis. Antibodies to ribonucleoproteins are characteristic of mixed
connective tissue disease. Anticollagen antibodies are detected in patients with
rheumatoid arthritis, and anti-PM-Sd has been detected in some patients with
overlapping features of myositis and scleroderma.

Rheumatology:Question 64
The correct answer is C
Educational Objectives
Recognize the importance of bladder and bowel dysfunction as an indication of spinal
cord corn promise.
Critique
This woman with new-onset systemic lupus erythematosus (SLE) may have
transverse myelitis of the spinal cord producing urinary retention. Transverse
myelitis may be more likely to occur in SLE when antiphospholipid antibodies are
present. Examination for a distended bladder is of primary importance at this time. If
an enlarged bladder is suggested by abdominal palpation or percussion,
catheterization of the bladder is indicated. If catheterization provides confirmation,
emergency MRI of the dorsal and lumbar sections of the spinal cord is indicated, and
if evidence of transverse myelitis is found, high-dose intravenous corticosteroids
should be given.
Urinalysis and culture are indicated on a subsequently obtained urine sample, with
special attention to the cellular elements, leukocytes (concern for infection or
interstitial nephritis induced by nonsteroidal anti-inflammatory drugs), or
erythrocytes/casts (indicative of glomerulitis).
Because she is alert and not mentally clouded or stuporous, evaluation of her brain
and central nervous system can be postponed. Although her temperature is slightly
elevated, she does not appear septic, and evaluation for possible bladder distention
takes precedence over a lumbar puncture at this time.
Eye ground examination is essential in patients with systemic lupus erythematosus
to rule out vasculitis (cotton wool exudates). Her alert status and normal blood
pressure make it unlikely that papilledema indicating increased intracranial pressure
would be found.
Total hemolytic complement determination is appropriate in any patient with newly
diagnosed, active systemic lupus erythematosus. This value could be low related to
consumption of complement components secondary to immune complex deposition,
particularly in the skin or kidneys, or to heritable deficiencies of C2 or C4.

Rheumatology:Question 65
The correct answer is C
Educational Objectives
Recognize that radiographic evidence of a loosened prosthesis does not indicate
whether the cause is infectious or mechanical.
Critique
It is difficult to know whether prosthesis loosening detected by radiography is due to
mechanical factors or to infection, especially when a patient is afebrile and does not
have leukocytosis. It is not unusual for older patients to have normal peripheral
blood leukocyte counts with infection. It is crucial to obtain joint fluid for Gram stain
and culture, and the procedure is best done under imaging. For that reason, referral
to an interventional radiologist would be appropriate.
All other management options are secondary to direct examination of joint fluid. The
erythrocyte sedimentation rate of 44 mm/h is not abnormal in an 81-year-old,
because erythrocyte sedimentation rate tends to increase with age.
In a recent study in which radiographs of 18,486 hip arthroplasties were reviewed, it
was shown that prosthesis loosening could be diagnosed correctly only by
radiography, not by physical examination in patients who present with hip pain after
total hip joint replacement. Asymptomatic patients did not have loosening.
Therefore, routine periodic radiography in asymptomatic patients with hip joint
replacements is unnecessary.

Rheumatology:Question 66
The correct answer is B
Educational Objectives
Recognize the frequency of zidovudine-induced myositis.
Critique
Simvastatin, zidovudine, cocaine, HIV, and alcohol can cause muscle abnormalities,
but only zidovudine is associated with characteristic changes in mitochondrial
histology.

Rheumatology:Question 67
The correct answer is C
Educational Objectives
Recognize the association of inflammatory bowel disease with spondyloarthropathy.
Critique
The physician should pursue other possible causes of what appears to be ankylosing
spondylitis in this patient. Although ankylosing spondylitis is an entirely consistent
diagnosis for this patient, a high erythrocyte sedimentation rate is not generally seen
in this disease. When the erythrocyte sedimentation rate or C-reactive protein level
is high in patients like this one, the focus first should be on gastrointestinal disease
or cutaneous problems.
If this patient were found to have inflammatory bowel disease, it might be
appropriate to use methylprednisolone or its equivalent at some time in the course of
therapy, but not until a firm diagnosis has been made. Sulfasalazine is appropriate
therapy for ankylosing spondylitis, but again, not until a firm diagnosis has been
attained. Blood loss or anemia of chronic disease would be a much more likely cause
of anemia than hemolysis in this clinical situation.
Recent studies have shown that sacroiliitis in Crohn’s disease is an isolated
phenomenon unrelated to HLA-B27, but that it may evolve into classical spinal
ankylosis in susceptible subjects. HLA-B27 determinations are not usually helpful in
clinical diagnosis and should rarely be ordered.
It is of interest that the clinical indications for therapy with anti-tumor necrosis factor
(anti-TNF) have been extended to include severe and progressive
spondyloarthropathies; indications for anti-TNF use may be particularly strong in
patients with Crohns disease and spondylitis.

Rheumatology:Question 68
The correct answer is C
Educational Objectives
Treat moderately severe disseminated gonococcal infection.
Critique
The history of migratory polyarthritis, the skin lesion, the tenosynovitis, and the
settling of arthritis in one joint are characteristic of disseminated gonococcal
infection. The systemic features (fever, chills) indicate a moderately severe course
that requires prompt, aggressive treatment.
Sexual histories, like drinking histories, are notoriously unreliable, and one cannot
place great stock in them. A menstrual history is helpful because women are more
vulnerable to disseminated gonococcal infection during menses and pregnancy.
Because of the prevalence of penicillin-resistant Neisseria, initial treatment should be
ceftriaxone until cultures and sensitivities are known.
Although nonsteroidal anti-inflammatory drugs and prednisone would reduce
inflammation, they would simply mask the problem, which would persist and lead to
joint destruction. If pain persists as the infection is treated, analgesics
(acetaminophen or codeine) can be prescribed. Prednisone might reduce host
defense.

Rheumatology:Question 69
The correct answer is C
Educational Objectives
Recognize predictors of outcome in adults with inflammatory polymyositis.
Critique
The presence of dysphagia predicts a complicated and difficult course in myositis;
none of the other variables does so. Myositis-specific antibodies are associated with
interstitial lung disease, and their absence is a good prognostic indicator.

Rheumatology:Question 70
The correct answer is B
Educational Objectives
Recognize the diagnostic importance of muscle weakness when it is associated with
polyarthritis.
Critique
Painless proximal muscle weakness associated with a proliferative synovitis suggests
either polymyositis or mixed connective tissue disease. An elevated serum creatine
kinase level would confirm the presence of an inflammatory myopathy. If the anti-
ribonucleoprotein component of extractable nuclear antigen antibody testing were
positive, the appropriate diagnosis would be mixed connective tissue disease. If
antiribonucleoprotein were negative, ordering a test for anti-Jo-i antibody would be
appropriate to help confirm a diagnosis of polymyositis. Recent studies have shown
that urinary creatine kinase is significantly elevated in patients with
polymyositis/dermatomyositis compared with controls.
Muscle disease can occur in rheumatoid arthritis. In addition to progressive atrophy
from disuse, in severe cases, inflammatory myositis can accompany the joint
disease. This patient is not taking statins; if he had been, a statin-associated
myopathy should also have been considered.
Until a diagnosis is more defined, baseline radiographs are not necessary. In the
presence of severe proliferative synovitis, hydroxychloroquine and ibuprofen are
inadequate therapy. Methotrexate in combination with hydroxychloroquine is more
appropriate. Modern therapy for polyarthritis and myositis does not include
prolonged bed rest, which leads to loss of muscle tone and bone loss; moderate rest
with defined stretching and isometric exercises is much more appropriate.
Rarely is proliferative synovitis or proximal muscle weakness symptomatic of the
musculoskeletal syndrome associated with hepatitis C virus infection. Palpable
purpura linked to cryoglobulinemia is more likely.

Rheumatology:Question 71
The correct answer is C
Educational Objectives
Recognize that an occult infection can cause chronic knee joint pain and swelling.
Critique
A finding of monoarthritis always suggests infection. The chronic nature of the
patient’s knee problem, the absence of osteoarthritis, and absence of mechanical
derangement by imaging studies suggest a process such as mycobacterial or fungal
infection. Therefore, arthroscopy is not indicated.
Tuberculosis is a leading cause of chronic monoarthritis in many parts of the world.
Synovial infection occurs by hematogenous seeding. The tuberculin skin test is
positive in 75% of patients, and rheumatoid factor is often present in chronic
infection. The plain film radiograph does not suggest osteonecrosis, which would
more likely show a line of bone lysis. Mycobacteria migrate and adhere to synovium
as they do to pleura; therefore, they are often absent in synovial or pleural fluid.
Synovial fluid analysis is nonspecific and does not help with the diagnosis. Acid-fast
stains of the synovium are usually negative, but histologic features of granulomatous
synovitis and positive cultures lead to a definitive diagnosis in nearly 95% of cases.

Rheumatology:Question 72
The correct answer is C
Educational Objectives
Understand the safety profile of hydroxychloroquine.
Critique
Although the antimalarial agents have an excellent safety profile, as many as half of
treated patients experience adverse events. Rash and gastrointestinal problems
(heartburn, nausea, vomiting, diarrhea) are most common. Most of these problems
remit spontaneously or with a reduction in the dosage.
Hydroxychloroquine and chloroquine can cause three types of ocular toxicity. Blurred
vision may occur early in the course of treatment because of defects in
accommodation or convergence (probably a central neural effect). Corneal deposition
of drug can cause blurred vision that may be associated with halos around lights;
this kind of symptom must be differentiated from glaucoma, especially in patients
treated with corticosteroids. These problems are benign and reversible and usually
remit with continued administration of drug.
Retinal toxicity is much more serious and can cause permanent loss of vision.
Retinopathy is rare, and almost always due to administration of doses greater than
published safe levels. How often to schedule ophthalmologic monitoring is
controversial. It has been considered prudent for patients to see an ophthalmologist
at least every 6 months so that early reversible changes in the retina can be
detected. The recommendation of the American Academy of Ophthalmology is for
annual eye examinations.

Rheumatology:Question 73
The correct answer is A
Educational Objectives
Diagnose hemochromatosis underlying osteoarthritis.
Critique
It is common to miss the diagnosis of hemochromatosis as the underlying cause of
osteoarthritis. It should be ruled out whenever a patient presents with osteoarthritis
of joints other than the spine, hips, and knee. Radiographs of joints before
arthroplasty often reveal chondrocalcinosis. The second and third
metacarpophalangeal joints (MCPJ) are often more symptomatic and involved with
osteoarthritis than are other MCPJ. A serologic test for saturated iron-binding
capacity should lead to more definitive testing to estimate the excess iron load.
Hemochromatosis is also associated with diabetes mellitus, impotence, and
congestive heart failure as iron is deposited in these organs.
Rheumatoid factor could be positive in hemochromatosis as well as in other liver
diseases, and would not help in diagnosis. A fluorescent antinuclear antibody titer
would not be expected to be positive. Radiographs of the shoulders that have been
replaced by prostheses would not show chondrocalcinosis, whereas radiographs of
the MCPJ, elbows, knees, or ankle that have not been surgically treated in this
patient would be likely to show linear calcification in articular or fibrocartilage. Tests
for cardiac function would possibly show abnormal results in this patient, and serum
testosterone would be expected to be low, but these findings would not lead to
definitive diagnoses.
Hereditary hemochromatosis also can masquerade as rheumatoid arthritis unless
careful physical examinations are done that reveal hard joint deformities rather than
soft synovial proliferation.

Rheumatology:Question 74
The correct answer is C
Educational Objectives
Diagnose arthritis due to parvovirus Big.
Critique
This patient has arthritis associated with parvovirus Bi9, which tends to occur in
adults exposed to schoolchildren. The macular rash in adults is indistinct and can
occur anywhere, in contrast to the “slapped cheeks rash seen in children.
Arthritis associated with parvovirus Bi9 is usually of acute onset and symmetrical.
The acute symptoms usually improve in 2 weeks, but arthralgias can last for several
months. Rarely, arthritis can persist for years and resemble rheumatoid arthritis,
requiring treatment with anti-inflammatory agents.
A positive result on an antinuclear antibody or rheumatoid factor test can be
associated with parvovirus Bi9 arthritis. The virus does not, however, cause lupus or
rheumatoid arthritis. Many people have lgG antibodies to parvovirus Big, but the
presence of 1gM antibodies suggests a recent infection. Radiography would be
unlikely to show any abnormalities after only 2 weeks. Prednisone is used sometimes
to suppress chronic joint inflammation of unknown cause; in this case, the short
history and the clues leading to suspicion of parvovirus would preclude the use of
prednisone.

Rheumatology:Question 75
The correct answer is B
Educational Objectives
Recognize that fibromyalgia can coexist with other disorders.
Critique
Although it appears that this woman has a fibromyalgia syndrome (FMS), the
unilateral knee pain is inconsistent with FMS. Patients with FMS often complain so
much about their generalized pain that these symptoms take precedence over
symptoms from other disorders. The lack of morning stiffness, the diffuseness of the
pain in the knee, and the completely normal knee examination should focus attention
on the hip. It is not unusual for hip pain associated with osteoarthritis to be referred
to the knee. Therefore, this patient should have radiographs of the hip.
It is common for FMS to exist concurrently (and be exacerbated by) other connective
tissue problems, including osteoarthritis, rheumatoid arthritis, and other diffuse
connective tissue diseases. Celecoxib would provide some symptomatic relief,
perhaps, but it is important that a diagnosis be made first. Cushioned shoes might
be helpful, as would isometric knee exercises, after diagnosis. Injection of the knee
joint with corticosteroids should be done only when there is a definite inflammation
within the joint and other possibilities have been ruled out.

Rheumatology:Question 76
The correct answer is C
Educational Objectives
Recognize a treatable adverse event due to methotrexate.
Critique
Pulmonary reactions to methotrexate appear to be more common in rheumatoid
arthritis than in other diseases. The initial response should be to stop the
methotrexate. The predominant symptom, dyspnea, is usually present for about 3
weeks before the diagnosis is made. Cough due to methotrexate-induced irritation of
the airways can occur without other symptoms and usually is not associated with
interstitial lung disease.
Pulmonary reactions to methotrexate usually occur within the first 18 months of
treatment. Mild elevations of serum transaminases are common in patients taking
methotrexate for rheumatoid arthritis, often in association with weekend partying,
and by themselves are not cause for alarm. Recurrent elevations in serum
transaminases or low serum albumin levels (all monitored every 8 weeks) would lead
to consideration of liver biopsy. It is important to do microbiology studies in order to
exclude infection.
Most patients do well with discontinuation of methotrexate and administration of
corticosteroids. Chronic interstitial lung disease with dyspnea has been reported,
however. The wrist pain is likely due to her known rheumatoid arthritis; therefore,
radiographs would not be helpful. Methotrexate-induced pneumonitis or alveolitis can
be treated with prednisone, but first the diagnosis must be confirmed.

Rheumatology:Question 77
The correct answer is A
Educational Objectives
Recognize the association of rheumatoid arthritis with popliteal cysts and the
symptoms of a ruptured cyst.
Critique
Administration of enoxaparin would be contraindicated. This patient is at high risk for
a ruptured popliteal cyst into the calf: She has had rheumatoid arthritis for more
than a decade, has had active synovitis of the knees, and has been engaged in mild
activity (hiking). Rupture of a popliteal cyst into the calf can mimic exactly deep
venous thrombosis, but administration of anticoagulant to a patient with a ruptured
popliteal cyst can lead to extensive hemorrhage.
None of the other choices would be contraindicated. Aspiration of the fluid in the left
knee would be appropriate to rule out infection in this moderately
immunocompromised patient who is taking a tumor necrosis factor-α inhibitor.
Hydrocodone bitartrate would give pain relief and not interfere with coagulation.
Intravenous corticosteroids would help diminish systemic inflammation but should
not be used if there is a high degree of suspicion for infection. After infection within
the joint has been ruled out, intra-articular corticosteroid is appropriate therapy for
this condition.
The proper sequence with such a patient is to perform an ultrasound examination of
the calf. Ultrasound delineates rupture or herniation of popliteal cysts (hypoechoic
space behind the calf muscles), and combined with Doppler testing, can help rule out
thrombophlebitis.

You might also like